Section 9 Nephrology
SECTION 9 Nephrology
648
CHAPTER 32
649
Acid-Base Disorders
650 Case: An 18-year-old man with polyuria A previously healthy 18-year-old man presents to the emergency
department with progressive nausea, vomiting, and abdominal pain. volume. Over the past day he has developed abdominal pain and exHeart rate is 134 beats per minute, respiratory rate is 32 breaths per acutely ill with deep and rapid breathing. There is a fruity odor on his 82Serum glucose is 600(mg/dL, sodium (Na+) 126 mEq/L, chloride (Cl-) Several weeks ago he noticed an increase in urinary frequency and nausea, worsening to the point of intense vomiting. He has perienced weight loss of 10 pounds over the past few weeks. minute, and blood pressure is 94/58 mm Hg. The patient appears breath. The abdomen is diffusely tender to palpation. mEq/L, bicarbonate ) 12 mEq/L, blood urea nitrogen (BUN) 34 mg/dL, and osmolality 305 mOsm/kg (reference range 275- 295 mOsm/kg). Arterial blood gas measurement shows a pH of 7.29 co and a partial pressure of carbon dioxide (Pa 2 ) of 26 mm Hg. What acid-base disturbance(s) is present in this patient?
651
652
653
654
Metabolic Acidosis
What are the 2 general subtypes of metabolic acidosis?
How is anion gap calculated? What anion gap value is considered elevated? Calculated anion gap should always be corrected for what common condition?
The subtypes of metabolic acidosis are nonâanion gap (non-AG) metabolic acidosis and anion gap (AG) metabolic acidosis.
AG =[Na ] â([Cl ] +[ ]) + â The reference range for AG varies by laboratory. However, in general, â¨AG >12 mEq/L is considered elevated.4 Calculated AG should always be corrected for hypoalbuminemia, as albumin is a significant unmeasured anion. For every 1 g/dL decrease in serum albumin concentration, 2.5 mEq/L should be added to the calculated AG. Hypophosphatemia may also affect the AG calculation to a smaller degree.3
What conditions are associated with low anion gap? What alternate term is used to describe non-anion gap (or normal anion gap) metabolic acidosis?
Low AG can be observed in conditions associated with high levels of cations, such as paraproteinemia in multiple myeloma, hypercalcemia, hypermagnesemia, and lithium toxicity.3 Non-AG metabolic acidosis is also known as hyperchloremic metabolic acidosis. When is removed from the blood (eg, through the Gastrointestinal [GI] tract), electroneutrality is maintained with a proportional rise in Cl , which also has a negative charge. Because the sum of â serum and Clâ is part of the formula for AG, there is no net change in the calculated AG (Figure 32-3).
FIGURE 32-3 Balance of anions and cations in the development of metabolic acidosis. The left panel shows the normal state with the sum of plus unmeasured anions (predominantly albumin) making up the gap. The middle panel shows that a decrease in accompanied by a
How do the lungs maintain acid-base homeostasis in the setting of metabolic acidosis? When lung function is normal, which formula can be used to predict Paco2 in the setting of metabolic acidosis? In the setting of metabolic acidosis, when measured Paco2 is higher than that predicted by the compensation formula, what additional acid-base disorder must be present? In the setting of metabolic acidosis, when measured Paco2 is lower than that predicted by the compensation formula, what additional acid-base disorder must be present?
corresponding increase in Clâ results in no change to the anion gap. The right panel shows development of acidosis through the generation or retention of H+ with a different anion (eg, lactate) leading to an Increased anion gap. (From Rennke HG, Denker BM. Renal Pathophysiology: The Essentials. 4th ed. Philadelphia, PA: Lippincott Williams & Wilkins; 2014.)
In the setting of metabolic acidosis, compensatory hyperventilation increases removal of the volatile acid CO2 (see Figure 32-1). When functioning normally, the kidneys also help correct acidemia by excreting acid and reabsorbing . Wintersâ formula can be used to calculate the expected Paco2 resulting from respiratory compensation in the setting of metabolic acidosis.3,5Predicted Paco2 =(1.5 Ă[ ]) +8 Âą 2 In the setting of metabolic acidosis, if measured Paco2 is higher than that predicted by the compensation formula, then a concomitant respiratory acidosis is present.
In the setting of metabolic acidosis, if measured Paco2 is lower than that predicted by the compensation formula, then a concomitant respiratory alkalosis is present.
655
NonâAnion Gap Metabolic Acidosis
What are the causes of non-anion gap metabolic acidosis?
656
657
658
Anion Gap Metabolic Acidosis
What is the mechanism of Increased anion gap in anion gap metabolic acidosis?
What is the delta anion gap (ÎAG)?
What is the delta anion gap in a patient with serum Na+ of 140 mEq/L, Clâ of 100 mEq/L, and of 8 mEq/L? How is serum affected by pure anion gap metabolic acidosis?
What is the predicted serum in a patient with pure anion gap metabolic acidosis if the anion gap is 28? How can the predicted serum be used to determine whether the anion gap metabolic acidosis is pure or mixed with other metabolic disorders?
AG metabolic acidosis results from acids that dissociate to produce a hydrogen ion and conjugate base. For example, lactic acid (C3H5O3) dissociates into H+ and lactate ( ), its conjugate base. The hydrogen ions consume , while the negatively charged conjugate base rises proportionally, maintaining electroneutrality. Consequently, serum is Decreased but Clâ remains unchanged. The increase in unmeasured anions (ie, the conjugate base) is reflected by an increase in the calculated AG (see Figure 32- 3). The ÎAG is the difference between the calculated AG and the upper limit of normal for AG (ie, 12 mEq/L). It should be calculated in patients with AG metabolic acidosis to determine whether the AG metabolic acidosis is pure or mixed with other metabolic disorders (ie, concomitant non-AG metabolic acidosis or concomitant metabolic alkalosis).12 ÎAG =32 â12 =20
In pure AG metabolic acidosis, the decrease in serum should be roughly the same as the ÎAG (ie, ratio of ÎAG to Î is roughly 1:1). Some processes result in a ratio very close to 1:1 (eg, diabetic ketoacidosis, early lactic acidosis), whereas other processes result in slightly different ratios (eg, late lactic acidosis results in a ratio of 1.6:1). The serum can therefore be predicted to a close approximation.3Predicted serum =24 âÎAG Âą 5 mEq/L Predicted serum =24 â(28 â12) Âą 5 =8 Âą 5 mEq/L (ie, 3-13 mEq/L)
Measured serum should be compared with predicted serum . If measured serum is within the range of values predicted, then the AG metabolic acidosis is pure. If measured serum is higher than the upper limit predicted, then there is another process producing excess (ie, concomitant metabolic alkalosis). If measured serum is lower than the lower limit predicted, then there is another process reducing (ie, concomitant non-AG metabolic acidosis).
What are the causes of anion gap metabolic acidosis?
A 58-year-old woman with atrial fibrillation presents with acute-onset severe abdominal pain with rigidity, guarding, and rebound tenderness and is found to have AG metabolic acidosis. Associated with asterixis. These acids can be detected on urine dipstick. A 34-year-old man with a history of alcohol dependence presents with blurry vision and AG metabolic acidosis.
What are the 2 general types of lactic acidosis?
What is the mechanism of anion gap metabolic acidosis in uremia?
Lactic acidosis from cardioembolic ischemia of the Gastrointestinal tract.
Uremia. Ketoacidosis. Methanol toxicity.
The 2 types of lactic acidosis are type A (oxygen delivery to the tissues is impaired) and type B (oxygen delivery to the tissues is not impaired). Common causes of type A lactic acidosis include hypovolemic shock (eg, hemorrhage), cardiogenic shock (eg, myocardial infarction), and distributive shock (eg, sepsis). Common causes of type B lactic acidosis include medications (eg, metformin), toxins (eg, alcohol), and malignancy (eg, lymphoma). Mild to moderate acute and chronic kidney disease are associated with non-AG metabolic acidosis when there is impaired acid excretion but relatively preserved glomerular filtration rate. In the setting of uremia or advanced chronic kidney disease, a significant reduction in glomerular filtration leads to the retention of unmeasured anions (eg, phosphate, sulfate, urate), resulting in AG metabolic acidosis.9
659
common in patients with type 1 or nonadherence to insulin therapy, infection, and ischemia (eg, myocardial infarction). The type 2 diabetes mellitus? cornerstones of management include isotonic fluid resuscitation, insulin administration, and electrolyte replacement as needed. What is the significance of ketone During periods of starvation, the brain switches from a reliance on glucose to ketone bodies as bodies during periods of starvation? the primary fuel for metabolism. This shift decreases the need for gluconeogenesis, which conserves protein, helping the body to survive for extended periods of time.10 What is the timing of alcoholic Alcoholic ketoacidosis typically develops in patients with a recent (hours to days) but not active ketoacidosis in relation to the alcohol binge. Ethanol inhibits lipolysis, so significant generation of ketone bodies occurs only consumption of alcohol? when ethanol levels begin to fall.11 Which medications are associated with Medications associated with AG metabolic acidosis include salicylates (eg, aspirin) and anion gap metabolic acidosis? acetaminophen. In addition, numerous medications can cause type B lactic acidosis (eg, isoniazid, linezolid, highly active antiretroviral therapy).12 Which metabolic abnormality can be a Elevated serum osmolal gap can be a clue to toxic alcohol ingestion. Serum osmolal gap is the clue to the presence of ethanol, difference between measured serum osmolality and calculated serum osmolality (normal is ethylene glycol, or methanol toxicity? <10 mOsm/kg).Calculated serum osmolality =(2 Ă[Na ]) +([glucose]/18) â¨+([BUN]/2.8)In the + + above formula, [Na ] is measured in mEq/L or mmol/L, and [glucose] and [BUN] are measured in mg/dL.3 In addition to hemodialysis, what can Methanol and ethylene glycol can cause central nervous system depression but are otherwise be used to treat methanol and ethylene relatively innocuous. However, toxic metabolites are generated when these parent alcohols are glycol poisoning? oxidized by alcohol dehydrogenase. Fomepizole or ethanol can be used to inhibit alcohol dehydrogenase, thereby preventing the generation of toxic metabolites.13
660
Respiratory Acidosis What is the basic mechanism of respiratory acidosis? How do the kidneys maintain acid-base homeostasis in the setting of persistent respiratory acidosis?
Which formula can be used to predict serum in the setting of acute respiratory acidosis? Which formula can be used to predict serum in the setting of chronic respiratory acidosis?
In the setting of respiratory acidosis, when measured serum is higher than that predicted by the compensation formula, what additional acid-base disorder must be present? In the setting of respiratory acidosis, when measured serum is lower than that predicted by the compensation formula, what additional acid-base disorder must be present?
Respiratory acidosis is caused by hypoventilation and associated hypercarbia. The kidneys respond to persistent respiratory acidosis by increasing reabsorption in the proximal tubule and increasing H+ excretion as titratable acid and (Figure 32-5).
FIGURE 32-5 Renal tubular hydrogen ion (H+) excretion. Drugs. 10thled. Philadelphia, PA:lLippincott Williams & Wilkins; 2014.) (From Al dredge BK, et al. App ied Therapeutics: The Clinical Use of
In the setting of acute respiratory acidosis, for every 10 mm Hg increase in Paco2 above 40 mm Hg, serum should increase by 1 mEq/L.3 In the setting of chronic respiratory acidosis, for every 10 mm Hg increase in Paco2 above 40 mm Hg, serum should increase by 4 to 5 mEq/L.3 In the setting of respiratory acidosis, if measured serum is higher than that predicted by the compensation formula, then a concomitant metabolic alkalosis is present. In the setting of respiratory acidosis, if measured serum is lower than that predicted by the compensation formula, then a concomitant metabolic acidosis is present.
What are the causes of respiratory acidosis?
A general decrease in respiratory drive. Difficulty getting air out of the lungs and airways. Excess alveolar ventilation relative to pulmonary capillary perfusion (ie, V/Q >1). Diaphragmatic weakness. Anatomic abnormalities that can be congenital or acquired.
What are the causes of central nervous system depression? What are the causes of airway obstruction? What are the causes of Increased dead space?
Which noninvasive bedside pulmonary function test can be used to gauge severity and progress in a patient with neuromuscular weakness? Which thoracic cage disorders can lead to
Central nervous system (CNS) depression. Airway obstruction. Increased dead space. Neuromuscular disease. Thoracic cage disorders.
Major causes of CNS depression include medications (eg, narcotics, benzodiazepines), stroke, trauma, encephalitis, central sleep apnea, and hypothermia. Major causes of airway obstruction include chronic obstructive pulmonary disease, asthma, obstructive sleep apnea, laryngospasm, tracheomalacia, and foreign body aspiration. Major causes of Increased dead space include shallow breathing, pulmonary embolism, Decreased pulmonary capillary perfusion (eg, from systemic hypotension, pulmonary hypertension, or pulmonary capillaritis), emphysema, and positive pressure ventilation (see chapter 46, Hypoxemia). Neuromuscular weakness can be evaluated with negative inspiratory force (NIF, or maximal inspiratory pressure [MIP]).
Thoracic cage disorders that can lead to hypoventilation include pectus excavatum,
661
662
Respiratory Alkalosis
What is the basic mechanism of respiratory alkalosis? Respiratory alkalosis is caused by hyperventilation and associated hypocarbia. How do the kidneys maintain acid-base homeostasis in the setting of The kidneys respond to persistent respiratory alkalosis by persistent respiratory alkalosis? decreasing reabsorption in the proximal tubule and decreasing H+ excretion as titratable acid and . Which formula can be used to predict serum in the setting of In the setting of acute respiratory alkalosis, for every 10 mm Hg acute respiratory alkalosis? decrease in Paco2 below 40 mm Hg, serum should decrease by 2 mEq/L.3 Which formula can be used to predict serum in the setting of In the setting of chronic respiratory alkalosis, for every 10 mm Hg chronic respiratory alkalosis? decrease in Paco2 below 40 mm Hg, serum should decrease by 4 to 5 mEq/L.3 In the setting of respiratory alkalosis, when measured serum is In the setting of respiratory alkalosis, if measured serum is higher than that predicted by the compensation formula, what higher than that predicted by the compensation formula, then a additional acid-base disorder must be present? concomitant metabolic alkalosis is present. In the setting of respiratory alkalosis, when measured serum is In the setting of respiratory alkalosis, if measured serum is lower than that predicted by the compensation formula, what lower than that predicted by the compensation formula, then a additional acid-base disorder must be present? concomitant metabolic acidosis is present.
What are the causes of respiratory alkalosis?
An 18-year-old student develops respiratory alkalosis before taking a math final. Between 37 and 32 BC, a Chinese official warned travelers en route to Afghanistan about âthe Little Headache Mountainâ and âthe Great Headache Mountain,â where âmenâs bodies become feverish, they lose color, and they are attacked with headache and vomiting.â Nine months of respiratory alkalosis. Decreased oxygen delivery to the tissues in a patient with normal cardiac output and normal oxygen hemoglobin saturation. Hyperventilation and respiratory alkalosis can be the first sign of this infectious process associated with end-organ dysfunction and Decreased systemic vascular resistance. Look for spider angiomas on the chest. A 45-year-old man develops respiratory alkalosis after starting treatment for smoking cessation.
Which central nervous system disorders are associated with hyperventilation? Why is hypoxemia associated with respiratory alkalosis?
While respiratory alkalosis occurs early on, which acid-base disorder eventually predominates in sepsis?
What is the most likely mechanism of respiratory alkalosis in the setting of cirrhosis?
Hyperventilation related to anxiety. Hypoxemia related to high altitude. The Chinese official was most likely describing acute mountain sickness.14,15
Pregnancy. Anemia.
Sepsis.
Cirrhosis. Nicotine therapy.
CNS disorders that can cause hyperventilation include anxiety, pain, fever, psychosis, trauma, stroke, infection (eg, meningitis, encephalitis), and space-occupying lesions (eg, tumor). Hypoxemia stimulates hyperventilation via the hypoxic ventilatory response. Mechanisms of hypoxemia include reduced inspired oxygen (eg, high altitude), Increased dead space ventilation (eg, pulmonary embolism), physiologic shunt (eg, pneumonia), diffusion impairment (eg, interstitial lung disease), and anatomic shunt (eg, atrial septal defect) (see chapter 46, Hypoxemia). These conditions tend to progress to respiratory acidosis when severe or when there is respiratory muscle fatigue.14,16 The predominant acid-base disorder of sepsis is AG metabolic acidosis â¨(ie, lactic acidosis).
Progesterone, which is metabolized in the liver and therefore elevated in the serum of patients with cirrhosis, can stimulate hyperventilation via progesterone receptors in the CNS. Estradiol, which is also elevated in the serum of patients with cirrhosis, may
663
When does respiratory alkalosis develop during the course of pregnancy?
Which medications are associated with respiratory alkalosis?
receptors.14 Minute ventilation begins to increase within the first weeks of pregnancy and is about 50% higher than that in nonpregnant women. However, because of renal compensation, pH during pregnancy is close to normal.17 Medications that can cause respiratory alkalosis include salicylates, nicotine, catecholamines, quetiapine, xanthines (eg, theophylline), progesterone, and medroxyprogesterone acetate.18,19
664
Metabolic Alkalosis
What 2 pathophysiologic conditions must be present for metabolic alkalosis to occur?
How do the lungs maintain acid-base homeostasis in the setting of metabolic alkalosis? Which formula can be used to predict the Paco2 value in the setting of metabolic alkalosis? In the setting of metabolic alkalosis, when measured Paco2 is higher than that predicted by the compensation formula, what additional acid-base disorder must be present? In the setting of metabolic alkalosis, when measured Paco2 is lower than that predicted by the compensation formula, what additional acid-base disorder must be present?
For metabolic alkalosis to occur, there must be (1) generation of metabolic alkalosis via the addition of new (either loss of acid or gain of alkali), and (2) impairment in the ability of the kidney to correct the alkalosis (usually a result of Decreased effective arterial volume, which augments reclamation in the kidney).20 In the setting of metabolic alkalosis, compensatory hypoventilation decreases removal of the volatile acid CO2. When functioning normally, the kidneys also help correct alkalemia by increasing excretion and decreasing acid excretion. The following formula can be used to calculate the expected Paco2 resulting from respiratory compensation in the setting of metabolic alkalosis3:Predicted Paco2 =0.7 Ă([ ] â24) +40 Âą 2 In the setting of metabolic alkalosis, if measured Paco2 is higher than that predicted by the compensation formula, then a concomitant respiratory acidosis is present.
In the setting of metabolic alkalosis, if measured Paco2 is lower than that predicted by the compensation formula, then a concomitant respiratory alkalosis is present.
What are the causes of metabolic alkalosis?
Metabolic alkalosis on a cruise ship. Loss of bicarbonate-free fluid. Iatrogenic polyuria. An electrolyte abnormality. Hypertension, hypokalemia, metabolic alkalosis, and an adrenal tumor. A 42-year-old woman with central obesity, moon facies, and thin skin. These 2 inherited conditions mimic the effects of loop and thiazide diuretics, respectively. A 45-year-old man with peptic ulcer disease presents with hypercalcemia and metabolic alkalosis. Correction of a respiratory process.
What are the causes of H+ loss from the Gastrointestinal tract? What are the main mechanisms by which the metabolic alkalosis of contraction alkalosis is maintained? What is the main mechanism of metabolic alkalosis in loop and thiazide diuretics, and Bartterâs and Gitelmanâs syndromes?
Gastrointestinal loss of H+ related to vomiting from viral gastroenteritis. Contraction alkalosis. Diuretic medications. Hypokalemia. Primary hyperaldosteronism.
Cushingâs syndrome.
Bartterâs and Gitelmanâs syndromes.
Exogenous alkali.
Post-hypercapnia.
Causes of H+ loss from the GI tract include vomiting, nasogastric lavage, and some cases of diarrhea (eg, villous adenoma, laxative abuse).21 The metabolic alkalosis of contraction alkalosis is maintained as a result â¨of Decreased effective arterial blood volume by the following mechanisms: â¨(1) Decrease in glomerular filtration rate reduces the filtered load of , (2) proximal tubule reabsorption of is enhanced, and (3) Decreased Clâ delivery to the cortical collecting tubule impairs secretion.20 Loop and thiazide diuretics and Bartterâs and Gitelmanâs syndromes result in elevated renin-angiotensin- aldosterone levels and Increased delivery of Na+ and H2O to the distal nephron, both of which increase urinary H+ secretion, causing metabolic alkalosis. In addition, these conditions often result in contraction alkalosis through the loss of bicarbonate-free fluid, further contributing to metabolic alkalosis.20
665
What are the mechanisms of metabolic alkalosis related to hypokalemia?
What is the mechanism of metabolic alkalosis related to primary hyperaldosteronism?
What are the causes of nonaldosterone mineralocorticoid excess? Why are blood products a potential source of exogenous alkali? What is the mechanism of metabolic alkalosis that follows a period of persistent hypoventilation?
Hypokalemia can generate and maintain metabolic aklalosis. In the setting of hypokalemia, there is movement of potassium from the intracellular to extracellular fluid compartment, with resultant movement of H+ into the cells to maintain electroneutrality, which increases extracellular pH. In addition, hypokalemia results in Decreased excretion and Increased H+ excretion in the kidney.20 Aldosterone acts in the distal nephron to increase Na+ reabsorption in exchange for H+ secretion, augmenting renal acid loss. Coexistent hypertension and hypokalemia can be a clue to the presence of primary hyperaldosteronism. Secondary hyperaldosteronism, with the exception of that generated by diuretics (or Bartterâs and Gitelmanâs), does not generate metabolic alkalosis because Increased delivery of Na+ and H2O to the distal nephron does not occur.20 Causes of nonaldosterone mineralocorticoid excess include Cushingâs syndrome, Liddle syndrome, and exogenous mineralocorticoids. Blood products can be anticoagulated with citrate salts, which are converted in the body to sodium bicarbonate. The kidneys compensate for hypercapnia by decreasing excretion of . When Paco2 is corrected, transient metabolic alkalosis will ensue before the kidneys excrete the extra .
666 Case Summary nausea, abdominal pain, and vomiting and is foundito have multiple A previously healthy 18-year-old man presents w th polyuria, metabolic abnormalities.
What acid-base disturbance(s) is present in this patient? Primary anion gap metabolic acidosis with appropriate respiratory compensation and concomitant metabolic alkalosis.
667
Bonus Questions
What areithe steps to determining the primary acid-base disorder n this case? (Recall serum Na 126 mEq/L, Cl + â 82 mEq/L, HCO3â12 mEq/L, serum osmolality 305 mOsm/kg, pH 7.29, and Paco 26 mm Hg.) 2 What type of metabolic acidosis is present in this case? Is there appropriate respiratory compensation for the metabolic acidosis in this case?
Is there another metabolic disorder in this case?
What is the most likely cause of the anion gap metabolic What is the significance of the deep and rapid breathing What is the most likely cause of the concomitant metabolic acidosis in this case? described in this case? alkalosis in this case? What is the serum osmolal gap in this case? What is the true serum Na+ in this case?
What are the main tenets of treating DKA?
The blood pH in this case indicates acidemia, which could result from either metabolic or respiratory acidosis. Low serum indicates that metabolic acidosis is the principal condition.
To determine the type of metabolic acidosis (non-AG or AG), the AG must be calculated. In this case, AG =126 â(82 +12) =32 mEq/L, which is elevated (normal is â¤12 mEq/L). 2 2 2 2 Using Wintersâ formula, expected Paco =(1.5 Ă12) +8 Âą 2 =26 Âą 2 mm Hg. â¨The Paco in this case is 26 mm Hg, indicating appropriate compensation. If Paco were >28 mm Hg, there would be insufficient respiratory compensation (ie, concomitant respiratory acidosis); if Paco were <24 mm Hg, there would be more respiratory compensation than expected (ie, concomitant respiratory alkalosis). The ÎAG =32 â12 =20 mEq/L. In a pure AG metabolic acidosis, serum would be expected to decrease by roughly the same amount from 24 to 4 mEq/L. Using the formula (predicted serum =24 âÎAG Âą 5 mEq/L), predicted serum =24 â20 Âą 5 =4 Âą 5 mEq/L. The serum in this case is >9 mEq/L, indicating concomitant metabolic alkalosis. Given the history of polyuria and the markedly elevated serum glucose level in this case, the most likely cause of AG metabolic acidosis is DKA secondary to a new diagnosis of type 1 diabetes mellitus. The deep and rapid breathing (Kussmaulâs respirations) describediin this case is associated with DKA, and reflects the compensatory hyperventilation that occurs in response to metabol c acidosis. Based on the history in this case, GI loss of H+from vomiting is the most likely explanation for the concomitant metabolic alkalosis. Calculated/serum osmolality =(126 Ă2) +(600/18) +(34/2.8) =297 mOsm/kg. The measured osmolality in this case is 305 mOsm kg. The osmolal gap =305 â297 =8 mOsm/kg (normal is <10 mOsm/kg). Measured serum Na+must be corrected for hyperglycemia. To do this, for every 100 mg/dL increase in glucose above 100 mg/dL, 1.6 to 2.4 mEq/L should be added to the measured Na . In this case, using a correctional factor of 2.0 mEq/L, the following should be added to the measured Na : ([600â100]/100) Ă2.0 =10. Corrected serum Na is + + 126 +10 =136 mEq/L. The cornerstones of managing DKA include crystalloid intravenous infusion for rehydration, insulin infusion to decrease the blood glucose level, and electrolyte replacement as needed.
668 Key Points
Acidemia is defined by.arterial pH <7.35. Normal arterial pH is 7 4 Âą 0.05. Alkalemia is defined by arterial pH >7.45.
Acidemia and alkalemia are caused by metabolic and respiratory acid-base disorders.
History and physical examination are critical components of the Primary metabolic acidosis is characterized by the combination investigation of any acid-base disturbance. of acidemia and low serum . ofPrimary respiratory acidosis is characterized by the combination coPrimary respiratory alkalosis is characterized by the acidemia and elevated Pa co 2 . mbination of alkalemia and low Pa co 2 . Primary metabolic alkalosis is characterized by the combination acThe subtypes of metabolic acidosistare non-AG metabolic of alkalemia and elevated serum . idosis (AG â¤12 mEq/L) and AG me abolic acidosis (AG
12 mEq/L). d In cases of AG metabolic acidosis, ÎAG should be calculated to The lungs rapidly compensate for metabolic acid-base disorders. etermine whether or not there is a concomitant metabolic disorder. The kidneys slowly compensate for persistent respiratory acid- base disorders. The adequacy o compensat on can be determined in any
primary acid-basefdisorder byiusing disorder-specific formulas.
669
References 1. Reddy P. Clinical approach to renal tubular acidosis in adult patients. Int J Clin Pract. 2011;65(3):350-360.
- Berne RML, Levy MN. Physiology. 4th ed. St. Louis, Missouri: Mosby, Inc.; 1998.
- Berend K, de.Vries AP, Gans RO.;Physiological approach to assessment of acid-base disturbances N Engl J Med. 2014 371(15):1434-1445.
-
Costanzo L. Physiology. 4th ed. Philadelphia, PA: Lippincott Williams & Wilkins; 2007. 5. Albert MS,iDell RB, Winters RW. Quantitative displacement of acid-base equilibrium 6. Ringerâs resuscitation on hemodynamics, metabolic responses, and coagulation in pigs in metabol c acidosis. Ann Intern Med. 1967;66(2):312-322. Martini WZ, Cortez DS, Dubick MA. Comparisons of normal saline and lactated after severe hemorrhagic shock. Scand J Trauma Resusc Emerg Med. 2013;21:86. Widmer B, Gerhardt RE, Harrington JT, Cohen JJ. Serum electrolyte and acid base
-
composition. The)influence of graded degrees of chronic renal failure. Arch Intern Med. 1979;139(10 :1099-1102.
-
Van der Aa F, Joniau S, Van Den Branden M, Van Poppel H. Metabolic changes after urinary diversion. Adv Urol. 2011;2011:764325.
-
Warnock DG. Uremic acidosis. Kidney Int. 1988;34(2):278-287.
- Cahill GF Jr. Starvation in man. N Engl J Med. 1970;282(12):668-675.
- Noor NM, Basavaraju.K, Sharpstone D. Alcoholic ketoacidosis: a case report and review of the literature Oxf Med Case Reports. 2016;2016(3):31-33.
- Pham AQ, Xu LH, Moe OW. Drug-induced metabolic acidosis. F1000Res. 2015;4.
-
Jacobsen D, McMartin KE. Antidotes for methanol and ethylene glycol poisoning. J Toxicol Clin Toxicol. 1997;35(2):127-143.
-
Lustik SJ, Chhibber AK, Kolano JW, et al. The hyperventilation.of cirrhosis: 15. Rennie D. The great breathlessness mountains. JAMA. 1986;256(1):81-82. progesterone and estradiol effects. Hepatology. 1997;25(1):55-58
- Moroz VA, Butrov AV. Mechanisms of compensation.of disorders of acid-base status of the blood in patients with chronic anemia. Ter Arkh 1985;57(7):74-79.
-
Huch R. Maternal hyperventilation and the fetus. J Perinat Med. 1986;14(1):3-17. 18. Fukuhara Y, Kaneko T, Orita Y. Drug-induced acid-base disorders. Nihon Rinsho. 1992;50(9):2231-2236.
-
Palmer BF. Evaluation and treatment of respiratory alkalosis. Am J Kidney Dis. 2012;60(5):834-838.
-
Palmer BF, Alpern RJ. Metabolic alkalosis. J Am Soc Nephrol. 1997;8(9):1462-1469.
- Perez GO, Oster)JR, Rogers A. Acid-base disturbances in Gastrointestinal disease. Dig Dis Sci. 1987;32(9 :1033-1043.
670
CHAPTER 33
671
Acute Kidney Injury
672 Case: A 73-year-old man with blue toes disease, type 2 diabetes mellitus, and peripheral artery disease istery A 73-year-old man with a history of hypertension, coronary ar evaluated in the clinic for skin changes 10 days after undergoing femoropopliteal bypass surgery. The patient had been doing well at
home after the procedure until he noticed a new rash over his legs and Blood pressure is 174/78 mm Hg. There is a lacy erythematous bluish discoloration of his toes. macular rash over the thighs (Figure 33-1) as well as bluish discoloration of the toes bilaterally (Figure 33-2). Dorsalis pedis pulses are palpable.
FIGURE 33-1 (Courtesy of Peter D. Sullivan, MD.)
FIGURE 33-2 (Courtesy of Lawrence B. Stack, MD.)
BlPeripheral white blood cell count is 5.8 K/ÂľL with 14%ieosinophils. Na ood urea nitrogen (BUN) is 58 mg/dL, and serum creat nine is 4.1 mg/dL. The fractional excretion of sodium (FE ) is 4.8%. Microscopic examination of urine sediment is notable for pyuria,
including eosinophils. There are no red blood cells or cellular casts. Renal ultrasound is unremarkable.
What is the most likely cause of Acute Kidney Injury in this patient?
What is Acute Kidney Injury (AKI)? What is the range of
AKI is the rapid development (within hours to days) of renal excretory dysfunction, characterized by the accumulation of the products of nitrogen metabolism (ie, urea and creatinine), Decreased urine output, or both.1 BUN is dependent on multiple factors, including protein intake, endogenous protein catabolism, intravascular
673
normal for blood urea fluid volume status, hepatic urea synthesis, and kidney function. This is reflected in the wide normal range nitrogen levels? reported by laboratories (eg, 5-20 mg/dL).2 Is the blood urea Elevated BUN can result from high-protein diet, antianabolic medications (eg, glucocorticoids, tetracyclines), nitrogen level a catabolic processes (eg, fever, infection), low effective arterial blood volume, and upper Gastrointestinal bleeding. reliable reflection of Decreased BUN can result from low protein diet, malnutrition, and impaired hepatic function. Given all of these renal function? influential factors, serum creatinine concentration is a more reliable predictor of renal function than BUN.2 What is normal serum Creatinine is produced and released by muscle without significant short-term (day-to-day) variation. Serum creatinine creatinine levels are therefore dependent on muscle mass, which is dependent on nutritional status, age, gender, concentration? and ethnicity. For the average adult man, the normal range for serum creatinine is 0.6 to 1.2 mg/dL; for the average adult woman, it is approximately 0.5 to 1.1 mg/dL. Normal serum creatinine in individuals with muscle mass above or below average (eg, body builders, elderly, malnourished patients) may be outside of those ranges.2 What is the AKI is defined by any of the following: (1) increase in serum creatinine by âĽ0.3 mg/dL within 48 hours, (2) increase biochemical in serum creatinine concentration to âĽ1.5 times baseline, known or presumed to have occurred within the prior laboratory definition 7 days, or (3) urine volume <0.5 mL/kg per hour for 6 hours.3 of Acute Kidney Injury? What is the expected In the setting of anuric kidney injury, serum creatinine will increase 1 to 2 mg/dL per day. Higher rates of rise can rise in serum be seen in patients with extreme catabolic states (2-3 mg/dL per day) or in those with crush injury and creatinine in a 24- rhabdomyolysis (>3 mg/dL per day).4,5 hour period in the average patient with anuric kidney injury? What risk factors are Risk factors for AKI include preexisting kidney disease (most significant), advanced age, diabetes mellitus, and associated with the black race.6 development of Acute Kidney Injury? What are the Symptoms are often absent in patients with AKI but, depending on the underlying cause may include Decreased symptoms of Acute urine output, peripheral edema, gross hematuria, dyspnea, and symptoms of uremia if present (eg, nausea). Kidney Injury? What are the physical Patients with AKI often do not have specific physical findings but, depending on the underlying cause, there may findings of Acute be hypertension, peripheral edema, elevated jugular venous pressure (JVP), inspiratory rales, cutaneous findings Kidney Injury? associated with specific diagnoses (eg, palpable purpura in patients with small vessel vasculitis), and findings of uremia if present (eg, pericardial friction rub in patients with uremic pericarditis). What are the sequelae Sequelae of severe AKI include electrolyte derangements (eg, hyperkalemia), hypervolemia, metabolic acidosis, of severe Acute and uremia. Kidney Injury? When should renal Renal replacement therapy should be considered in patients with anuria (negligible urine output for 6 hours), replacement therapy severe oliguria (urine output <200 mL over 12 hours), severe hyperkalemia (potassium >6.5 mEq/L), severe be considered for metabolic acidosis (pH <7.2 with normal or low Paco ), hypervolemia (particularly if pulmonary edema is present), 2 patients with Acute severe azotemia, or clinical sequelae of uremia (eg, pericarditis).1 Kidney Injury? What is the natural In some patients, AKI leads to new chronic kidney disease (CKD); those with preexisting CKD are at risk for history of Acute disease progression after an episode of AKI, including the development of end-stage renal disease (ESRD). Renal Kidney Injury? recovery occurs in many patients with AKI. However, even when renal function initially recovers, patients who experience AKI are at Increased risk for later development of CKD.6 What is the definition CKD is defined as the presence of kidney damage (eg, albuminuria) or Decreased kidney function (ie, GFR of chronic kidney <60 mL/min/1.73 m2) for âĽ3 months.7 disease? Which causes of AKI from any cause can progress to CKD. However, there is a higher rate of progression in patients with acute Acute Kidney Injury tubular necrosis (ATN) compared with those without ATN.6 can lead to chronic kidney disease? What are the 3 AKI can be caused by prerenal, intrarenal (ie, intrinsic), or postrenal processes. anatomic categories of Acute Kidney Injury?
What structures differentiate the 3 anatomic categories of Acute Kidney Injury?
Prerenal causes of AKI affect structures up to and including the afferent arteriole (without injuring the renal parenchyma). Intrarenal causes of AKI affect the renal parenchyma (including glomeruli, interstitium, renal tubules, and blood vessels), even if the inciting cause involves a prerenal structure (eg, cardiogenic shock causing ATN). Postrenal causes of AKI include any process that obstructs the flow of urine from the kidneys to the urethra (Figure 33-3).
674
FIGURE 33-3 Acute Kidney Injury can be related to prerenal, intrarenal, or postrenal processes. (Adapted from Porth CM. Essentials of Pathophysiology. 3rd ed. Philadelphia, PA: Lippincott Williams & Wilkins; 2010.)
675
Prerenal Acute Kidney Injury
What is the general mechanism of prerenal Acute Kidney Injury?
Prerenal AKI generally occurs as a result of Decreased renal blood flow.
What biochemical laboratory data can be suggestive of prerenal Acute Kidney Injury?
What are the characteristics of urine sediment in the setting of prerenal Acute Kidney Injury?
A serum BUN:creatinine ratio >20:1 can occur when there is Increased urea reabsorption in the proximal tubule triggered by low effective arterial blood volume. A FENa of <1% in patients with oliguria is also suggestive of a prerenal etiology. However, FENa can be influenced by diuretic medications. In patients with recent or active diuretic exposure, the fractional excretion of urea can be used instead (a value <35% is consistent with prerenal AKI).8â10 The urine sediment of prerenal AKI is typically bland, but hyaline casts may be present (Figure 33-4).11
FIGURE 33-4 Hyaline cast (arrow) surrounded by red blood cells. Note the low refractive index of the cast (400Ă). (From Mundt LA, Shanahan K. Graffâs Textbook of Urinalysis and Body Fluids. 3rd ed. Philadelphia, PA: Wolters Kluwer; 2016.)
What are the causes of prerenal Acute Kidney Injury?
A patient with a recent diagnosis of heart failure is started on dietary restrictions and diuretic therapy and subsequently presents with weight loss of 10 pounds, low JVP, and AKI. A previously healthy 56-year-old woman with a recent back strain develops prerenal AKI after taking escalating doses of over-the- counter analgesic medication. An increase in total body volume with Decreased effective arterial blood volume and an S3 gallop (see Figure 4-3). A condition characterized by impaired systemic vascular resistance and reduced renal perfusion. A 63-year-old man with small cell lung cancer develops abdominal pain, confusion, and prerenal AKI. A decrease in effective arterial blood volume as a result of splanchnic vasodilation. A 42-year-old woman with marked hypertension and a bruit over the abdomen. Reduced renal blood flow as a result of compression of the renal veins.
Which of the following patterns of jugular venous pressure (JVP), cardiac output (CO), and systemic vascular resistance (SVR) are characteristic of hypovolemia, sepsis, and cardiorenal syndrome?
Hypovolemia.
Nonsteroidal anti-inflammatory drugs (NSAIDs).
Cardiorenal syndrome.
Distributive shock (eg, sepsis).
Hypercalcemia.
Hepatorenal syndrome (HRS). Renal artery stenosis (RAS). Abdominal compartment syndrome.
Hypovolemia is associated with pattern C (low JVP, low CO, and high SVR); sepsis is associated with pattern B (low JVP, high CO, and low SVR); and cardiorenal syndrome is associated with pattern A (high JVP, low CO, and high SVR). Common causes of hypovolemia include poor oral intake, GI loss (eg, diarrhea, vomiting), diuretic use, hemorrhage, and insensible losses.
676
JVP CO SVR A â â â B â â â C â â â
What are the mechanisms of prerenal Acute Kidney Injury related to nonsteroidal anti-inflammatory drugs, angiotensin-converting enzyme inhibitors/angiotensin receptor blockers, and calcineurin inhibitors? What is the mechanism of prerenal Acute Kidney Injury related to hypercalcemia? What are the clinical differences between the 2 subtypes of hepatorenal syndrome?
What is the mechanism of Acute Kidney Injury in the setting of renal artery stenosis?
At what point does Increased abdominal pressure result in oliguria and anuria?
NSAIDs, ACE-I/ARBs, and CIs cause AKI as a result of reduced glomerular filtration pressure via various effects on the afferent and efferent arterioles. NSAIDs impair afferent arteriolar dilation; ACE-I/ARBs impair efferent arteriolar vasoconstriction; and CIs increase afferent arteriolar vasoconstriction.12 Hypercalcemia causes a reduction in glomerular filtration related to afferent arteriolar constriction. It also causes renal salt wasting, leading to hypovolemia.4 Type 1 HRS is rapid in onset with a doubling of serum creatinine to a level above 2.5 mg/dL in less than 2 weeks and is commonly associated with multiorgan failure. Type 2 HRS is characterized by a more indolent, stable course and is commonly associated with refractory ascites. Oliguria is more likely to occur in the setting of type 1 HRS.13 RAS is a chronic process and does not, in and of itself, cause AKI. However, the presence of RAS increases the risk of renal hypoperfusion, leading to AKI. For example, the development of AKI after the initiation of an ACE-I/ARB can be a clue to the diagnosis of RAS. Oliguria and anuria develop when intra-abdominal pressures reach approximately 15 mm Hg (20 cm H2O) and 30 mm Hg (41 cm H2O), respectively.14
677
Intrarenal Acute Kidney Injury
What are the general subcategories of intrarenal Acute Kidney Injury?
Intrarenal AKI can be caused by vascular disease, glomerulonephritis (GN), ATN, or acute interstitial nephritis (AIN) (Figure 33-5).
FIGURE 33-5 Cross-section of the kidney (A) and the nephron (B), illustrating the potential structures involved in intrarenal AKI. (From Carter PJ. Lippincottâs.Textbooklfor Nursing Assistants: A Humanistic Approach to Caregiving. 3rd ed Philade phia, PA: Wolters Kluwer Health; 2012.)
678
Vascular Causes of Intrarenal Acute Kidney Injury
What are the names of the arteries between the renal Renal artery â segmental artery â interlobar artery â arcuate artery and the afferent arteriole? lar causes of prartery â cortical radiate artery â afferent arteriole.nal blood flow Unlike the vascu erenal AKI that cause low re (eg, afferent arteriole constriction), the vascular causes of intrarenal AKI cause renal parenchymal disease.
What are the vascular causes of intrarenal Acute Kidney Injury?
Sudden flank pain and AKI in a patient with atrial fibrillation. Sudden flank pain and AKI in a patient with atherosclerotic RAS. Inflammation involving the walls of the renal artery. This condition usually follows arterial instrumentation. Thrombocytopenia, hemolytic anemia, and AKI. A 52-year-old woman with sinus disease, hemoptysis, AKI, and dysmorphic red blood cells (see Figure 34-4) and red blood cell casts on evaluation of urine sediment.
Renal artery embolism. Renal artery thrombosis. Large and medium vessel vasculitis. Atheroembolic renal disease. Thrombotic microangiopathy (TMA). Small vessel vasculitis related to granulomatosis with polyangiitis (GPA, or Wegenerâs granulomatosis).
What are the risk factors for Risk factors for renal artery embolism include atrial fibrillation, ischemic heart disease, renal artery embolism? cardiomyopathy, and valvular disease (eg, Infective endocarditis).15 What is the most common Severe flank pain is the most common presenting symptom of renal artery thrombosis. Onset is most presenting symptom in patients often sudden, but can be gradual in some cases, with maximum intensity reached after a few hours. with renal artery thrombosis? The pain is constant and may radiate to the lower quadrant or chest. Associated Gastrointestinal symptoms such as nausea and vomiting are common. AKI develops when the other kidney is diseased at baseline and cannot compensate for the abrupt increase in excretory load.16 Which types of vasculitis can The renal arteries can be affected by large vessel vasculitis (eg, giant cell arteritis, Takayasu arteritis) involve the renal artery? and medium vessel vasculitis (eg, polyarteritis nodosa, Kawasaki disease).17 What are the laboratory features Atheroembolic renal disease can present with peripheral and urinary eosinophilia and serum of atheroembolic renal disease? hypocomplementemia. Other laboratory abnormalities can indicate additional organ involvement (eg, hepatocellular liver injury suggests hepatic involvement).18 Which causes of thrombotic In addition to the primary TMA syndromes (eg, thrombotic thrombocytopenic purpura, hemolytic microangiopathy can present uremic syndrome, drug-induced TMA), other causes of TMA that can present with microangiopathic with Acute Kidney Injury? hemolytic anemia, thrombocytopenia, and AKI include disseminated intravascular coagulation, malignant hypertension, HELLP syndrome, and scleroderma renal crisis.19 Which small vessel vasculitides The small vessel vasculitides associated with intrarenal AKI include GPA, eosinophilic granulomatosis tend to be associated with with polyangiitis (EGPA, or Churg-Strauss syndrome), microscopic polyangiitis, and Henoch- intrarenal Acute Kidney Injury? SchĂśnlein purpura. These entities often cause GN.17 Glomerulonephritis is discussed in depth in chapter 34, Glomerular Disease.
679
680
Acute Tubular Necrosis
What is the mechanism of Acute Kidney Injury in patients with acute tubular necrosis? How common is acute tubular necrosis in hospitalized patients? What biochemical laboratory data can be suggestive of acute tubular necrosis? What are the characteristics of urine sediment in the setting of acute tubular necrosis?
Tubular dysfunction occurs as a result of injury to the tubular epithelial cells, usually ischemic or toxic in nature.20 ATN is the most common cause of AKI in the hospital (just under one-half of all cases), and is particularly prevalent in the intensive care unit (more than one-half of all cases).20 FENa >1% to 2% in patients with oliguric AKI is suggestive of ATN. FENa can be influenced by diuretic medications. In patients with recent or active diuretic exposure, the fractional excretion of urea can be used instead (a value >35%-50% is consistent with ATN).8,10 Examination of urine sediment in the setting of ATN typically reveals the presence of âmuddy brownâ granular casts (Figure 33-6). ATN is highly likely when at least 6 granular casts are seen.11
FIGURE 33-6 âMuddy brownâ cast (a type of granular cast). (From Mundt LA, Shanahan K. Graffâs Textbook of Urinalysis and Body Fluids. 3rd ed. Philadelphia, PA: Wolters Kluwer; 2016.)
What are the causes of acute tubular necrosis?
A 52-year-old man presents with crushing chest pain, hypotension, elevated JVP, and AKI with muddy brown casts on evaluation of urine sediment. A 23-year-old immunocompromised man develops ATN after starting treatment for cryptococcal meningitis. A 64-year-old man is hospitalized with diverticulitis and develops AKI 2 days after abdominal cross-sectional imaging is obtained. AKI develops after a crush injury. Anemia, elevated protein gap, low anion gap, and AKI.
What laboratory studies can be helpful in recognizing when prerenal Acute Kidney Injury has evolved
Prerenal spectrum resulting in renal ischemia (from cardiogenic shock).
Amphotericin B.
Contrast-induced nephropathy.
Pigment nephropathy. Protein injury from multiple myeloma.
In cases of ATN secondary to prerenal AKI, FENa can be <1%, making this test less useful. The absence or presence of granular casts on evaluation of urine sediment can be helpful. The absence of granular casts is associated with likelihood ratios of 4.5 for prerenal AKI and 0.2 for ATN. The presence of at least 6 granular casts is associated with a likelihood ratio of 10 for ATN and 0.10 for prerenal AKI. A fluid challenge can also
681
into acute tubular necrosis? What is the timing and prognosis of contrast- induced nephropathy? What medications are associated with acute tubular necrosis? In the setting of pigment nephropathy, what combination of findings on urine dipstick and urine microscopic analysis is classically seen? Why are pigment- and contrast-induced nephropathies sometimes associated with FENa <1%? What has been shown to improve renal recovery in patients with light chain cast nephropathy?
be helpful: Renal function recovery after volume repletion is consistent with prerenal AKI but not ATN.11 Contrast-induced nephropathy typically develops 24 to 48 hours after contrast exposure. Renal recovery occurs in most cases, with function returning to baseline in 7 to 10 days.21 Medications associated with ATN include NSAIDs, aminoglycosides, vancomycin, polymyxins, pentamidine, amphotericin B, foscarnet, tenofovir, cisplatin, and methotrexate.21 In the setting of pigment nephropathy, urine dipstick will be positive for blood in the absence of red blood cells on microscopic evaluation. The positive dipstick result is caused by the presence of pigment (myoglobin or hemoglobin). This combination of findings can be an important clue to the diagnosis.
In addition to causing toxicity to the renal tubules, pigment and contrast can induce constriction of the afferent arteriole, resulting in prerenal physiology.4
In patents with light chain cast nephropathy, there is a linear relationship between reduction in serum free light chain concentration (eg, via plasma exchange) and renal recovery.22
682
Acute Interstitial Nephritis
What is acute interstitial nephritis? What is the classic clinical triad of acute interstitial nephritis?
What are the characteristics of urine sediment in patients with acute interstitial nephritis?
What is the treatment for acute interstitial nephritis?
AIN is characterized by inflammation and edema within the renal interstitium, often associated with AKI.23 The classic clinical triad of AIN is fever, maculopapular rash, and peripheral eosinophilia. Although each component of the triad occurs commonly, the triad itself is present in a small percentage of patients with AIN overall (about 10%-15%). It may occur more frequently with certain etiologies of AIN (eg, methicillin- induced).23 Examination of urine sediment in the setting of AIN reveals leukocyturia and leukocyte casts in most cases (Figure 33-7). The significance of urine eosinophils is less clear.23
FIGURE 33-7 White blood cell cast (500Ă). (From Mundt LA, Shanahan K. Graffâs Textbook of Urinalysis and Body Fluids. 3rd ed. Philadelphia, PA: Wolters Kluwer; 2016.)
The cornerstone of treating AIN is addressing the underlying cause (in most cases, this is the removal of an offending medication). Still, renal function may not fully recover in a significant proportion of patients. Systemic glucocorticoids may be helpful in some cases.23
What are the causes of acute interstitial nephritis?
Responsible for more than 75% of the cases of AIN. Responsible for approximately 15% of cases of AIN; the medications used to treat these illnesses cause AIN more frequently. A 54-year-old black man with hilar lymphadenopathy (see Figure 21-4), hypercalcemia, and AKI. A combination of AIN and uveitis.
What medications are associated with acute interstitial nephritis?
When does acute interstitial nephritis usually develop in relation to exposure to a medication? Which bacterial and viral organisms are associated with acute interstitial nephritis?
What autoimmune diseases are
Medication.23 Acute interstitial nephritis associated with infection.23
Acute interstitial nephritis associated with sarcoidosis.
Tubulointerstitial nephritis and uveitis (TINU) syndrome.
Numerous medications can cause AIN, but the most common agents are NSAIDs and antibiotics (eg, penicillins, cephalosporins, ciprofloxacin, rifampicin, sulfonamides, vancomycin). Other common medications associated with AIN include allopurinol, acyclovir, famotidine, furosemide, omeprazole, and phenytoin.23 The average delay between exposure to a medication and the development of AIN is 7 to 10 days, but it can occur sooner, particularly if there is repeat exposure to an offending drug.23
Bacterial organisms associated with AIN include Brucella, Campylobacter, Escherichia coli, Legionella, Salmonella, Streptococcus, Staphylococcus, and Yersinia; viral organisms include cytomegalovirus, Epstein- Barr virus, hantavirus, and human immunodeficiency virus.23
Autoimmune diseases associated with AIN include sarcoidosis, SjĂśgrenâs syndrome, and systemic lupus
683
interstitial nephritis? What demographic of patients is most commonly affected by tubulointerstitial nephritis and uveitis syndrome?
TINU syndrome most frequently affects young women. It is characterized by the presence of uveitis, tubulointerstitial nephritis, and constitutional symptoms. Pathogenesis is unknown, but likely involves autoimmunity against elements common to the uveal tract and renal tubulointerstitium. Systemic glucocorticoids are the treatment of choice, and often reverse kidney injury.24
684
Postrenal Acute Kidney Injury
What is the mechanism of postrenal Acute Kidney Injury? What imaging findings are characteristic of postrenal Acute Kidney Injury? What bedside procedure can diagnose and treat some causes of postrenal Acute Kidney Injury? What is the prognosis of postrenal Acute Kidney Injury?
Postrenal causes of AKI obstruct the flow of urine from the kidneys to the outside world. This obstruction increases pressure within the tubules, resulting in a decrease in glomerular filtration rate. Hydronephrosis and dilated renal calyces, which can be identified with renal imaging, are characteristic of postrenal AKI. Obstruction of the bladder or structures distal to it can often be relieved with placement of a Foley catheter.
Renal recovery in patients with postrenal AKI is dependent on the duration and severity of obstruction. In patients with acute obstruction, complete recovery is generally achieved if the obstruction is relieved within 1 week; however, there is little chance for renal recovery if the duration of obstruction exceeds 12 weeks.25
What are the postrenal causes of Acute Kidney Injury?
A common condition found only in men. An asymmetric, hard, nodular prostate gland. To cause AKI in patients without preexisting kidney disease, this condition must be bilateral. A 28-year-old man who has been taking escalating doses of over-the-counter allergy medication presents with anuria and AKI. Patients with this common Endocrinopathy are at risk for the development of peripheral neuropathy, gastroparesis, and neurogenic bladder.
How common is benign prostatic hyperplasia in men?
What malignancies are associated with renal obstruction? Which imaging techniques are most useful in the evaluation of nephrolithiasis?
What other causes of mechanical obstruction can lead to postrenal Acute Kidney Injury? What classes of medications are associated with neurogenic bladder? What is the treatment for bladder dysfunction associated with diabetes mellitus?
What other causes of neurogenic bladder can lead to postrenal Acute Kidney Injury?
Benign prostatic hyperplasia (BPH). Prostate cancer. Nephrolithiasis.
Neurogenic bladder from anticholinergic medication.
Diabetes mellitus.
BPH increases in prevalence with age and is found in one-quarter of men who are in the sixth decade of life, one-third of those in the seventh decade of life, and one-half of all men older than 80 years of age.26 Malignancies associated with renal obstruction include prostate cancer, bladder cancer, renal cell carcinoma, transitional cell carcinoma of the collecting system, multiple myeloma, and metastases. Abdominal and pelvic imaging is critical in making the diagnosis. Ultrasonography and computed tomography imaging without contrast are the tests of choice in the evaluation of nephrolithiasis. Ultrasonography has the advantage of avoiding radiation exposure when used as the initial imaging technique. Despite the higher sensitivity of computed tomography imaging, there is no evidence that it improves outcomes when used as the initial imaging technique.27 Other causes of mechanical obstruction include urethral stricture, blood clot (particularly in patients who have recently undergone urinary tract instrumentation), crystal-induced intratubular obstruction (eg, acyclovir, methotrexate, protease inhibitors, ethylene glycol), and retroperitoneal fibrosis.28 Medications associated with neurogenic bladder include anticholinergics, narcotics, sedative hypnotics, antipsychotics, antidepressants, antispasmodics, and calcium channel blockers.29 Diabetes mellitus is associated with various types of bladder dysfunction, ranging from detrusor overactivity to poor emptying and bladder outlet obstruction. Treatment options for patients with urinary retention include behavioral modification (eg, voiding at regular intervals), pharmacologic therapy (eg, cholinergic agents), and catheterization to empty the bladder.30 Other causes of neurogenic bladder include stroke, multiple sclerosis, spinal cord injury, Parkinsonâs disease, and congenital disorders (eg, spina bifida, cerebral palsy).29
685
686 Case Summary A 73-year-old man with a history of vascular disease presents
10 days after femoropopliteal bypass surgery with new skin changes and is found to have AKI with peripheral and urinary eosinophilia.
What is the most likely cause of Acute Kidney Injury in this patient? Atheroembolic renal disease.
687
Bonus Questions
What is atheroembolic renal disease?
Whatiare the epidemiologic risk factors Which historical clues in this case are assoc ated with atheroembolic renal disease? suggestive of atheroembolic renal disease? Which clinical features in this case are suggestive of atheroembolic renal disease? What is the utility of the FE in this Na case? What is theiutility of the renal precipitated by what procedure?nly ultrasound n this case? Atheroembolism is most commo What is the treatment and prognosis of atheroembolic renal disease?
Atheroembolic renal disease occurs when atheromatous plaques of the large arteries (eg, aorta) arefdisrupted and embolize to the small renal 18 arteries and arterioles, causing acute occlusion and ischemia. AKI typically develops in a stepwise ashion in the weeks after plaque disruption. Risk factors for atheroembolic renal disease include older age (>60 years), male sex, hypertension, diabetes mellitus, vascular disease, cigarette smoking, and white race. 18 The history of vascular disease and recent intravascular procedure in this case are clues to the diagnosis of atheroembolic renal disease.
The skin findings (livedo reticularis and blue-colored toes [see Figures 33-1 and 33-2]), as well as the peripheral and urinary eosinophilia in this case are consistent with atheroembolic renal disease.
The FENa of >1% in this case argues against a prerenal cause of AKI. The renal ultrasound in this case does not show hydronephrosis or dilated renal calyces, arguing against postrenal AKI. Atheroembolism is most commonly associated with coronary angiography.18 There is no definitive treatment for atheroembolic renal disease other than treatment foriunderlying risk factors. Prognosis is generally poor: Up to one-half of patients develop dialysis-dependent kidney disease, and mortality is s gnificant at 1 year. 18
688 Key Points
AKI is the rapid development (within hours to days) of renal products of nitrogen metabolism (urea and creatinine), Decreased AKI can be caused by prerenal, intrarenal, or postrenal processes. excretory dysfunction, characterized by the accumulation of the urine output, or both. Prerenal AKI is caused by Decreased renal perfusion without injury to the renal parenchyma.
Intrarenal AKI is caused by injury to the renal parenchyma, even if Postrenal AKI is caused by obstruction of urine flow anywhere Prerenal AKI is associated with elevated serum BUN:creatinine Intrarenal AKI can be caused by vascular disease, GN, ATN, or Vascular causes of intrarenal AKI affect the medium- and small-GN is associated with dysmorphic red blood cells and red blood ATN is associated with FENa >1% to 2% and the presence of the inciting cause involves a prerenal structure. from the kidneys to the urethra. Na ratio (>20:1) and FE of <1%. AIN. sized arteries. cell casts on evaluation of urine sediment. granular casts on evaluation of urine sediment. AIN is associated with fever, rash, and peripheral and urinary Postrenal AKI is associated with hydronephrosis and dilated renal eosinophilia. calyces on imaging.
689
References 1. Bellomo R, Kellum JA, Ronco C. Acute Kidney Injury. Lancet. 2012;380(9843):756-766. 2. WalkertHK, Hall WD, Hurst JW, eds. Clinical Methods: The History, Physical, and Labora ory Examinations. 3rd ed. Boston: Butterworths; 1990. 2. KDIGO AKI Work Group. KDIGO clinical practice guidelines for Acute Kidney Injury. Kidney Int Suppl. 2012;17:1-138.
-
Abuelo JG, ed. Renal Failure Diagnosis & Treatment. Dordrecht, The Netherlands: Kluwer Academic Publishers; 1995.
-
Mosenifar Z, Hoo GWS, eds. Practical Pulmonary and Critical Care Medicine Disease Management. New York, NY: Taylor & Francis Group; 2006.
-
Chawla LS, Eggers PW, Star RA, Kimmel PL. Acute Kidney Injury and chronic kidney disease as interconnected syndromes. N Engl J Med. 2014;371(1):58-66.
-
KDIGO. Chapter 1: Definition and classification of CKD. Kidney Int Suppl. 2013;3:19.
- urea.in the differential diagnosis of acute renal failure. Kidney Int. 2002;62(6):2223-9. Dossetor JB. Creatininemia versus uremia. The relative significance of blood urea Carvounis CP, Nisar S, Guro-Razuman S. Significance of the fractional excretion of 2229 nitrogen and serum creatinine concentrations in azotemia. Ann Intern Med.
1966;65(6):1287-1299. 10. Steiner RW. Interpreting the fractional excretion of sodium. Am J Med. 1984;77(4):699- 702.
-
Perazella MA, Parikh CR. How can urine microscopy influence the differential diagnosis of AKI? Clin J Am Soc Nephrol. 2009;4(4):691-693.
-
Macedo E, Mehta RL. Prerenal failure: from old concepts to new paradigms. Curr Opin Crit Care. 2009;15(6):467-473.
-
Gines P, Schrier RW. Renal failure in cirrhosis. N Engl J Med. 2009;361(13):1279-1290. 14. Richards WO, Scovill W, Shin B, Reed W. Acute renal failure associated with Increased intra-abdominal pressure. Ann Surg. 1983;197(2):183-187.
-
Kansal S, Feldman M, Cooksey S, Patel S. Renal artery embolism: a case report and review. J Gen Intern Med. 2008;23(5):644-647.
-
Goodyear WE, Beard DE. Diagnosis and management of renal-artery thrombosis report of a case. N Engl J Med. 1947;237(10):355-358.
-
Jennette;JC, Falk RJ. The pathology of vasculitis involving the kidney. Am J Kidney 18. Scolari F, Ravani P. Atheroembolic renal disease. Lancet. 2010;375(9726):1650-1660. 19. George JN, Nester CM. Syndromes of thrombotic microangiopathy. N Engl J Med. Dis. 1994 24(1):130-141. 2014;371(7):654-66.
-
Gill N, Nally,JV Jr, Fatica RA. Renal failure secondary to acute tubular necrosis: epidemiology diagnosis, and management. Chest. 2005;128(4):2847-2863.
-
Pazhayattil GS, Shirali AC. Drug-induced impairment of renal function. Int J Nephrol Renovasc Dis. 2014;7:457-468.
-
Hutchison CA, Cockwell P, Stringer S, et al. Early reduction of serum-free;light chains associates with renal recovery in myeloma kidney. J Am Soc Nephrol. 2011 22(6):1129- 1136.
-
Praga M, Gonzalez E. Acute interstitial nephritis. Kidney Int. 2010;77(11):956-961.
690
- Sessa A, Meroni M, Battini G, Vigano G, Brambilla PL, Paties CT. Acute renal failure due to idiopathic tubulo-intestinal nephritis and uveitis: âTINU syndromeâ. Case
report and review of the literature. J Nephrol. 2000;13(5):377-380. 25after relief of complete unilateral ureteral obstruction of three monthsâ duration inction . Better OS, Arieff AI, Massry SG, Kleeman CR, Maxwell MH. Studies on renal fun man. Am J Med. 1973;54(2):234-240.
- McVary KT. BPH: epidemiology and comorbidities. Am J Manag Care. 2006;12(5 suppl):S122-S128. . Smith-Bindman R, Aubin C, Bailitz J, et al. Ultrasonography versus computed
27tomography for suspected nephrolithiasis. N Engl J Med. 2014;371(12):1100-1110. 28. Yarlagadda SG, Perazella MA. Drug-induced crystal nephropathy: an update. Expert Opin Drug Saf. 2008;7(2):147-158.
- Dorsher PT, McIntosh PM. Neurogenic bladder. Adv Urol. 2012;2012:816274. 30. Liu;G, Daneshgari F. Diabetic bladder dysfunction. Chin Med J (Engl). 2014 127(7):1357-1364.
691
CHAPTER 34
692
Glomerular Disease
693 Case: A 78-year-old man with hematuria A previously healthy 78-year-old man is admitted to the hospital cough, sore throat, and runny nose. Several days later he developedy involving his legs. After a few days, his urine turned reddish in color. thBlood pressure is 184/93 mm Hg. There is a palpable skin rash on with hematuria, skin rash, and abdominal pain. The patient reports recovering from an illness 1 to 2 weeks ago that was characterized b abdominal pain. This was followed by the onset of a skin rash The cough has resolved, and he does not complain of dyspnea. e lower extremities (Figure 34-1).
FIGURE 34-1 (Courtesy of Shahana F. Baig-Lewis, MD.)
Peripheral white blood cell count is normal with normal differential. blood cells/hpf. Serum antineutrophil cytoplasmic antibodies (ANCA) not identified. Serum complement levels are within normal limits. Serum creatinine is 2.9 mg/dL. Urine microscopy identifies 120 red and antiâglomerular basement membrane (anti-GBM) antibodies are Urine sediment findings are shown in Figure 34-2A and B.
FIGURE 34-2
(From McClatchey KD. Clinical Laboratory Medicine. 2nd ed. Philadelphia, PA: Lippincott Williams & Wilkins; 2002.)
What is the most likely diagnosis in this patient?
What is glomerular disease? What is the glomerulus?
Glomerular disease describes a heterogeneous group of conditions that damage various glomerular structures and produce characteristic clinical, physiologic, biochemical, and histologic manifestations. The glomerulus is a cluster of capillaries and associated mesangium at the proximal end of the nephron in the kidney. Bowmanâs capsule surrounds the glomerulus, and together they form the renal corpuscle, the basic filtration unit of the kidney (Figure 34-3).1
694
FIGURE 34-3 Anatomy of the glomerulus. A, The relationship of the glomerulus to Bowmanâs (urinary) space. B,
What are the effects of glomerular injury on the content of urine? What are the 2 general clinical syndromes of glomerular disease?
What are the characteristic urinary findings of nephrotic syndrome? What are the characteristic urinary findings of nephritic syndrome? What name is given to the group of inflammatory glomerular diseases that result in nephritic syndrome?
What is the role of renal biopsy in patients with nephrotic syndrome or glomerulonephritis?
The cellular anatomy of the glomerulus (cross-section). Note the relationships among capillary endothelial cells, basement membrane, and visceral epithelial cells. C, The glomerular membrane is composed of capillary endothelium, basement membrane, and visceral epithelial cells. Small soluble molecules cross the glomerular membrane from blood into Bowmanâs (urinary) space as glomerular filtrate (GF). In healthy patients, red blood cells and proteins are too large to cross. (McConnell TH. â¨Nature of Disease: Pathology for the Health Professions. 2nd ed. Philadelphia, PA: Wolters Kluwer Health; 2013.)
Glomerular injury results in impaired filtration. Consequently, constituents of blood that are normally excluded from the urinary space (eg, red blood cells [RBCs], white blood cells [WBCs], proteins) pass through the damaged glomerulus and are excreted in urine.2 Glomerular disease generally presents with either nephrotic syndrome or nephritic syndrome. Some diseases result in both nephrotic and nephritic syndromes, but most are associated with one or the other.
Nephrotic syndrome is characterized by proteinuria of at least 3.5 g/d. A minority of patients may also experience microscopic hematuria, but the urine sediment is typically bland. Glomerular diseases that manifest purely as nephrotic syndrome are noninflammatory in nature.3,4 Nephritic syndrome is characterized by active urine sediment consisting of dysmorphic RBCs, RBC casts, and occasionally WBCs and WBC casts. There can be varying degrees of proteinuria that are typically mild to moderate. Glomerular diseases that manifest as nephritic syndrome are inflammatory in nature.1 Glomerulonephritis (GN) refers to a group of inflammatory glomerular diseases that result in nephritic syndrome.
Renal biopsy includes routine examination of tissue under light, immunofluorescence, and electron microscopy; the patterns of glomerular injury identified by these techniques can often establish a diagnosis. The degree of severity and extent of active versus chronic (irreversible) changes can direct treatment and provide important prognostic information.2
695
Nephrotic Syndrome
What is the Nephrotic syndrome occurs when there is impairment of glomerular charge and size selectivity, which are pathophysiology of normally maintained by a combination of the glomerular basement membrane, endothelial cells, and nephrotic syndrome? epithelial cells (podocytes) (see Figure 34-3). The resultant increase in glomerular permeability allows large molecules such as albumin to escape into the urine.4 What are the symptoms of Symptoms of nephrotic syndrome may include peripheral edema (often anasarca), fatigue, dyspnea, and nephrotic syndrome? foamy urine. What are the physical Physical findings of nephrotic syndrome may include hypertension, generalized dependent pitting findings of nephrotic peripheral edema, ascites, and pleural effusions. Less common findings include abnormalities of the nail (eg, syndrome? Muehrckeâs lines), eruptive xanthomata, and xanthelasma. What are the general Nephrotic syndrome is characterized by proteinuria >3.5 g/d, serum albumin <2.5 g/dL, and hyperlipidemia laboratory features of (total cholesterol usually >180 mg/dL). Serum creatinine concentration can be variable in nephrotic nephrotic syndrome? syndrome.3 Which validated laboratory The measurement of protein and creatinine concentrations from a single, untimed (spot) sample of urine can calculation can effectively estimate daily proteinuria. The spot protein:creatinine ratio correlates with the protein content of a 24-hour determine the degree of urine collection (in grams per day).5 daily proteinuria with a single urine measurement? Why is nephrotic The mechanism of thrombophilia in nephrotic syndrome is incompletely understood but, in general, is syndrome associated with related to an imbalance between prothrombotic and antithrombotic factors. This imbalance occurs as a result thromboembolism? of the loss of anticoagulant proteins in the urine, including antithrombin and proteins C and S, and Increased production of procoagulant proteins such as fibrinogen. The most common sites of thrombosis in nephrotic syndrome are the renal veins, and the veins of the lower extremities.3 Why is nephrotic Patients with nephrotic syndrome develop infections (eg, cellulitis) more frequently than the normal syndrome associated with population. Mechanisms include low serum immunoglobulin G (IgG) concentration, reduced complement infection? activity, and diminished T-cell function.3
What pharmacologic agents are available to treat the proteinuria associated with nephrotic syndrome? What are the 2 general subcategories of nephrotic syndrome?
What are the differences between primary and secondary causes of nephrotic syndrome?
Angiotensin-converting enzyme inhibitors (ACE-I) or angiotensin receptor blockers (ARBs) are effective in decreasing proteinuria in patients with nephrotic syndrome and may slow disease progression.3
Nephrotic syndrome can be primary or secondary.
Primary glomerulonephropathies present with characteristic clinicopathologic patterns but are not related to known or identifiable systemic diseases or exposures (ie, they are idiopathic). Secondary glomerulonephropathies, which are associated with systemic diseases or exposures, tend to present with the clinicopathologic patterns of a particular type of primary glomerulonephropathy. Clues to a secondary cause include multiorgan involvement and other clinical manifestations associated with particular systemic diseases (eg, the malar rash of systemic lupus erythematosus [SLE]).
696
Primary Causes of Nephrotic Syndrome
What are the primary causes of nephrotic syndrome?
Because of the histologic distribution of this condition, there is a risk of inadvertently missing the involved tissue with biopsy. This entity is more frequently associated with thromboembolism compared with other causes of nephrotic syndrome. This entity is more common in children but is responsible for up to 15% of primary cases of nephrotic syndrome in adults.
Focal segmental glomerulosclerosis (FSGS).
Membranous nephropathy.3
Minimal change disease.6
Which ethnic group is at highest risk for focal FSGS is the most common cause of glomerular disease in black patients segmental glomerulosclerosis? (approximately one-half of cases), whereas membranous nephropathy is the most common cause in white patients.3 How frequently is membranous nephropathy Membranous nephropathy is primary (ie, idiopathic) in most cases. The remaining related to an underlying condition or exposure? cases are secondary to autoimmune disease (eg, SLE), infection (eg, hepatitis B virus), medication (eg, penicillamine), or malignancy (eg, colon cancer).7 Which pharmacologic agent is considered first- Systemic glucocorticoids are used as first-line treatment for minimal change disease line treatment for minimal change disease in in adults, with remission achieved in >80% of cases. However, relapses are common, adults? and these patients require repeated treatments, sometimes becoming dependent on or refractory to steroids.6,8 Which 2 primary glomerular diseases normally Membranoproliferative glomerulonephritis (MPGN) and immunoglobulin A (IgA) associated with nephritic syndrome can present nephropathy present with nephrotic syndrome in a small but significant number of with nephrotic syndrome in a small but cases. These entities are discussed later in this chapter. sigSecondary glomerulonephropathies tend to mirror the clinicopathologic nificant number of cases? patterns of a particular type of primary glomerulonephropathy (eg, SLE can present with a membranous nephropathy pattern) and must be ruled out before a diagnosis of primary (or idiopathic) glomerulonephropathy is given.
697
Secondary Causes of Nephrotic Syndrome
What are the secondary causes of nephrotic syndrome?
Kimmelstiel-Wilson lesions on histology. A 32-year-old woman in her 28th week of pregnancy develops hypertension, peripheral edema, and nephrotic-range proteinuria. A 56-year-old woman with rheumatoid arthritis develops nephrotic syndrome after starting a new therapy. A 23-year-old woman with a history of intravenous drug use presents with fever, holosystolic murmur at the apex with radiation to the axilla, positive blood cultures, and nephrotic-range proteinuria. A 22-year-old man with weight loss, night sweats, generalized lymphadenopathy, and nephrotic syndrome. An autoimmune disease capable of causing both nephrotic and nephritic syndromes. Diagnosed with Congo red staining and polarized light microscopy.
What clinical features suggest that the development of nephrotic syndrome in a diabetic patient may be related to a condition other than diabetes? What is the prognosis of nephrotic syndrome related to preeclampsia? What medications are associated with nephrotic syndrome? What infections are associated with nephrotic syndrome?
What malignancies are associated with nephrotic syndrome?
What are the clinical features of membranous lupus nephropathy (MLN)?
What are the renal
Diabetes mellitus. Preeclampsia.
Gold or penicillamine therapy.
Infective endocarditis.
Hodgkinâs lymphoma.
Systemic lupus erythematosus.
Amyloidosis.
The suspicion that nephrotic syndrome in diabetic patients may be related to another condition rises when any of the following features are present: nephrotic syndrome with normal renal function, the absence of retinopathy, rapid deterioration of renal function, active urine sediment, gross or microscopic hematuria, and short duration of diabetes.9
A return to normal renal function without evidence of ongoing proteinuria in the weeks to months after delivery is the rule in most patients with nephrotic syndrome related to preeclampsia. Prognosis for the fetus, on the other hand, is generally poor.10 Medications most commonly associated with nephrotic syndrome include nonsteroidal anti- inflammatory drugs, lithium, gold, penicillamine, captopril, and tamoxifen.3,4 Nephrotic syndrome can be associated with bacterial infection (eg, infectious endocarditis, syphilis), viral infection (eg, human immunodeficiency virus, hepatitis B virus, hepatitis C virus), protozoal infection (eg, malaria, toxoplasmosis), and helminthic infection (eg, schistosomiasis, filariasis).3,4 Malignancies most commonly associated with nephrotic syndrome include lymphoma, multiple myeloma, lung cancer, and renal cell carcinoma. The most common histologic patterns of glomerular disease in patients with malignancy-associated nephrotic syndrome are minimal change disease and membranous nephropathy.11 A little more than one-half of all patients with SLE will develop clinically evident kidney disease. Among them, 10% to 15% have MLN. Patients with MLN may not manifest the typical clinical and laboratory findings of SLE. Renal disease does not progress as aggressively as the proliferative forms of lupus nephritis, but a small percentage of patients with MLN will progress to end-stage renal disease. Patients with MLN are at risk for the complications of nephrotic syndrome (eg, thromboembolism).12 Acute or chronic renal insufficiency is present in up to one-half of patients with AL amyloidosis.
698
manifestations of immunoglobulin light chain (AL) amyloidosis? In addition to amyloidosis, what are the other 2 glomerular diseases associated with fibrillar deposits in the mesangium or glomerular basement membrane?
Proteinuria is common, occurring in most patients at presentation, with half of those cases being in the nephrotic range. Patients with nephrotic syndrome related to AL amyloidosis have a poor prognosis with median survival of 16 months. 13 In addition to amyloidosis, the other main glomerulonephropathies associated with fibrillar deposits in the glomeruli are fibrillary GN (more common) and immunotactoid glomerulopathy (less common). These entities can be distinguished from amyloidosis under the microscope because the fibrils are larger than those seen in amyloidosis and are Congo redânegative. Fibrillary GN and immunotactoid glomerulopathy are idiopathic in most cases but can develop secondary to systemic diseases (eg, multiple myeloma). Nephrotic syndrome is the most common clinical presentation, but many patients also have nephritic syndrome.14
699
Glomerulonephritis
What are the symptoms of glomerulonephritis? What are the physical findings of glomerulonephritis? What are the general laboratory findings of glomerulonephritis? What type of dysmorphic red blood cell is most specific for glomerulonephritis?
Patients with GN may experience hematuria and the symptoms of associated conditions such as Acute Kidney Injury (eg, oliguria, pleuritic chest pain from uremic pericarditis) nephrotic syndrome (eg, peripheral edema), and hypertension (eg, headache). Physical findings of GN may include hypertension, skin findings associated with specific underlying etiologies (eg, palpable purpura in patients with small vessel vasculitis), and findings of uremia if present (eg, pericardial friction rub in patients with uremic pericarditis). General laboratory findings of GN include features of the nephritic syndrome: urine sediment consisting of dysmorphic RBCs, RBC casts, and occasionally WBCs and WBC casts; varying degrees of proteinuria; and varying degrees of Acute Kidney Injury. Other laboratory studies can be indicative of specific underlying causes of GN (eg, serum cryoglobulins in the setting of cryoglobulinemic GN).1 The presence of acanthocytes, ring-shaped RBCs with vesicle-shaped protrusions (commonly described as âMickey Mouse earsâ), is most specific for the presence of GN (Figure 34-4). Acanthocyturia âĽ5% is associated with a sensitivity of about 50% and specificity of about 98% for GN.15
FIGURE 34-4 Dysmorphic red blood cells with vesicle-shaped protrusions (arrows), known as acanthocytes, are specific for glomerulonephritis. (Courtesy of Dr. Mark D. Okusa.)
What are the subcategories of glomerulonephritis based on serological testing?
The causes of GN can be separated into the following serologic patterns: positive ANCA, positive anti-GBM antibodies, low complement levels, and other.
It is important to recognize that the serologic patterns of GN are not 100% sensitive, and as such, etiologies that belong to these categories can present with a negative serologic workup in some cases. Renal biopsy is the single most definitive investigation into the cause of glomerular disease, including GN.
700
Glomerulonephritis Associated with ANCA
What does ANCA refer to?
In ANCA-positive patients, what are the 2 distinct patterns that may be seen on immunofluorescence microscopy?
What characteristic finding on immunofluorescence and electron microscopy distinguishes ANCA-associated glomerulonephritis from other forms of glomerulonephritis?
Antineutrophil cytoplasmic antibodies are a group of autoantibodies directed against proteins found in the cytoplasm of neutrophils, including proteinase 3 (PR3) and myeloperoxidase (MPO).16 When antibodies to PR3 are present, it results in a cytoplasmic pattern (c-ANCA). Antibodies to MPO are associated with a perinuclear pattern (p-ANCA). Enzyme-linked immunosorbent assays (ELISA) can specifically detect the presence of PR3 and MPO antibodies.16
The histologic hallmark of ANCA-associated GN is the paucity or lack of glomerular immune deposits (ie, pauci-immune).16
701
Glomerulonephritis Associated with c-ANCA
What disease causes glomerulonephritis associated with c-ANCA?
A 68-year-old woman with a history of sinusitis and saddle-nose deformity (see Figure 50-4) presents with hemoptysis, hematuria, and Acute Kidney Injury and is found to have positive c-ANCA titers.
Are all cases of granulomatosis with polyangiitis associated with ANCA?
How common is renal involvement in patients with granulomatosis with polyangiitis?
Granulomatosis with polyangiitis (GPA, or Wegenerâs granulomatosis).
ANCA is positive in up to 90% of cases of systemic GPA. The association is not as strong in patients with limited disease. Among those that are positive, most are c-ANCA. Despite this strong association, the absence of ANCA positivity does not exclude the diagnosis of GPA. Renal biopsy may be necessary to confirm the diagnosis.17 At the time of presentation, up to one-fifth of patients with GPA will have renal involvement. GN eventually develops in the vast majority of patients within the first 2 years. Treatment consists of an induction phase followed by maintenance therapy. Induction regimens for patients with GN usually consist of glucocorticoids in combination with cyclophosphamide or rituximab, which achieves remission in most patients. Methotrexate or azathioprine is usually used for maintenance therapy.1,17,18
702
Glomerulonephritis Associated with p-ANCA
What are the causes of glomerulonephritis associated with p-ANCA?
The absence of Microscopic polyangiitis (MPA). granulomas on histology is a distinguishing feature of this small vessel vasculitis. Think of this entity in Eosinophilic granulomatosis with polyangiitis (EGPA, or Churg-Strauss syndrome). patients with a history of asthma who present with GN.
Are all cases of ANCA is positive in up to 75% of patients with MPA. Among those that are positive, most are p-ANCA. Despite microscopic this strong association, the absence of ANCA positivity does not exclude the diagnosis of MPA. Renal biopsy polyangiitis associated may be necessary to confirm the diagnosis.19 with ANCA? How common is renal The vast majority of patients with MPA develop renal manifestations (>80%), ranging in severity from involvement in patients asymptomatic proteinuria to rapidly progressive GN. Induction regimens for patients with GN usually consist with microscopic of glucocorticoids in combination with cyclophosphamide or rituximab, which achieves remission in most polyangiitis? patients. Methotrexate or azathioprine is usually used for maintenance therapy.1,19 Are all cases of ANCA is positive in approximately one-half of all patients with EGPA but tends to be more common in those eosinophilic with GN. Among those that are positive, most are p-ANCA. The absence of ANCA positivity does not exclude granulomatosis with the diagnosis of EGPA. Renal biopsy may be necessary to confirm the diagnosis.20 polyangiitis associated with ANCA? How common is renal Approximately one-quarter of patients with EGPA develop renal manifestations, ranging in severity from involvement in patients asymptomatic urinary abnormalities (eg, microscopic hematuria) to end-stage renal disease. Induction regimens with eosinophilic for patients with GN usually consist of glucocorticoids either alone or in combination with cyclophosphamide, granulomatosis with which achieves remission in most patients. Azathioprine or methotrexate is usually used for maintenance polyangiitis? therapy.20
703
Glomerulonephritis Associated with Anti-GBM Antibodies
What disease causes glomerulonephritis associated with anti-GBM antibodies?
Aptly named.
Are all cases of anti-GBM disease associated with serum anti-GBM antibodies?
What is the hallmark of anti-GBM disease on immunofluorescence microscopy? What syndrome is characterized by the combination of pulmonary hemorrhage and glomerulonephritis related to anti- GBM disease?
Antiâglomerular basement membrane disease.
Using conventional methods, serum anti-GBM antibodies are present in up to 90% of patients with anti-GBM disease. Despite this strong association, the absence of anti-GBM antibodies does not exclude the diagnosis of anti-GBM disease. Renal biopsy may be necessary to confirm the diagnosis.21 Continuous linear deposition of immunoglobulin (usually IgG) along the glomerular basement membrane is the hallmark of anti-GBM disease on immunofluorescence microscopy.21 The combination of anti-GBM GN and pulmonary hemorrhage is known as Goodpasture syndrome. It is present in around one-half of patients with anti-GBM disease. In rare cases, pulmonary hemorrhage occurs in the absence of GN.21
704
Glomerulonephritis Associated with Low Serum Complement Levels The pattern of serum hypocomplementemia (ie, the differences in levels of C3 and C4) can be suggestive of particular etiologies.
What are the causes of glomerulonephritis associated with low serum complement levels?
Dark and scant urine following Post-streptococcal glomerulonephritis (PSGN). an illness characterized by sore throat, strawberry tongue, and a skin rash that began on the face and progressed to the upper trunk and extremities. The most common subtype of Membranoproliferative glomerulonephritis. this pattern of glomerular injury is characterized by the presence of subendothelial immune deposits on electron microscopy. A 26-year-old woman with renal Systemic lupus erythematosus. impairment, active urine sediment, the presence of antinuclear antibodies (ANA), antiâdouble-stranded deoxyribonucleic acid (anti-dsDNA) antibodies, and low serum complement levels. A 25-year-old woman with Infective endocarditis (IE). active intravenous drug use presents with dyspnea and fever and is found to have a decrescendo diastolic murmur over Erbâs point, painful nodules on the pulps of her fingers, and hematuria with dysmorphic RBCs. Chronic hepatitis C infection, Cryoglobulinemia. palpable purpura, and GN.
When does post-streptococcal glomerulonephritis usually develop during the course of streptococcal infection? What are the causes of membranoproliferative glomerulonephritis?
What is the timing of onset of lupus nephritis during the course of systemic lupus erythematosus?
PSGN typically occurs 7 to 10 days after upper respiratory tract infection and 2 to 4 weeks after skin infection. Prognosis is excellent in children, but there is a significant risk of chronic kidney disease or death in adults. The typical pattern of serum hypocomplementemia in PSGN is low C3 with normal C4 levels (C3 and C4 levels are both reduced in some patients).22 MPGN can occur as a primary (ie, idiopathic) or secondary glomerular disease, resulting from various conditions such as infection (eg, hepatitis C), autoimmune disease (eg, SLE), and plasma cell dyscrasia (eg, monoclonal gammopathy of unknown significance). The typical pattern of serum hypocomplementemia in MPGN depends on subtype. Complement-mediated MPGN usually presents with low C3 and normal C4 levels. Immune complexâmediated MPGN usually presents with normal or mildly low C3 and low C4 levels.23 Lupus nephritis is more common in younger patients. It can occur at any time during the course of the disease but most commonly presents within 1 year of diagnosis of SLE (it presents within 5 years of diagnosis in the vast majority of cases). The typical pattern of serum hypocomplementemia in lupus nephritis is reduction in both C3 and C4 levels; however, C4 levels are more often and more profoundly Decreased compared with C3.24,25
705
What is the differential diagnosis of renal injury in the setting of Infective endocarditis?
What are the main subtypes of cryoglobulinemia?
The main causes of renal injury related to IE include GN, renal infarction related to septic emboli, acute interstitial nephritis (AIN) related to antibiotic therapy (eg, β-lactams), and acute tubular necrosis (ATN) related to septic physiology or antibiotic therapy (eg, aminoglycosides). GN is typically an early manifestation of IE, occurring at the summit of the illness. Other causes of renal injury related to IE, such as antibiotic-associated AIN, tend to occur later in the course of the illness. The typical pattern of serum hypocomplementemia in IE-associated GN is low C3 with normal C4 levels (C3 and C4 levels are both reduced in some patients).26,27 Type I cryoglobulinemia (a monoclonal Ig, usually IgM or IgG) commonly occurs in the setting of hematologic malignancy (eg, WaldenstrÜm macroglobulinemia, multiple myeloma). Type II cryoglobulinemia (a mixture of polyclonal Ig with a monoclonal Ig, usually IgM) commonly occurs in the setting of chronic infection (eg, hepatitis C virus, human immunodeficiency virus). Type III cryoglobulinemia (a mixture of polyclonal Ig of all isotypes) commonly occurs in the setting of autoimmune disease (eg, SLE, SjÜgren syndrome) but can also occur with chronic infection (eg, hepatitis C virus). The typical pattern of serum hypocomplementemia in cryoglobulinemia-associated GN is markedly low C4 with normal or mild-to-moderately reduced C3 levels.28,29
706
Other Causes of Glomerulonephritis
What are the other causes of glomerulonephritis?
The most common primary glomerular disease worldwide. Abdominal pain, arthritis, palpable purpura, and GN.
What are the clinical features of immunoglobulin A nephropathy?
How common is glomerulonephritis in patients with Henoch-SchĂśnlein purpura?
Immunoglobulin A nephropathy.30
Henoch-SchĂśnlein purpura (HSP).
IgA nephropathy can present at any age but is most common in the second and third decades of life, with a predilection for white and Asian men. The 2 most common presentations of IgA nephropathy are (1) episodes of macroscopic hematuria that often coincide with or occur within 5 days of an upper respiratory tract infection (more common in patients <40 years of age), and (2) asymptomatic patients with abnormal urine sediment and proteinuria (more common in older patients). A minority of patients present with nephrotic syndrome or rapidly progressive GN. Among all patients with IgA nephropathy, up to 40% will eventually progress to end-stage renal disease. Glucocorticoids with or without other immunosuppressive agents may halt disease progression. In patients who progress to end-stage renal disease, kidney transplantation is the treatment of choice. IgA nephropathy recurs in a significant proportion of patients after transplantation.30 Approximately one-half of patients with HSP develop GN. The histologic features of HSP are identical to that of IgA nephropathy. The 2 conditions are distinguished by the occurrence of extrarenal manifestations in HSP, including palpable purpura, which is universally present.31
707 Case Summary upper respiratory tract infection and is found to have hypertension, nt A 78-year-old man presents with abdominal pain following a rece palpable purpura, Acute Kidney Injury, and hematuria.
What is the most likely diagnosis in this patient? Henoch-SchĂśnlein purpura.
708
indicate glomerular inflammation. Bonus Questions
What is the significance of case? the urine sediment in this What is the significance of respiratory tract infection What is the significance of the recent upper in this case? the skin finding in this case? Which procedure would b helpful in confirming the e SchĂśnlein purpura in this What is the treatment for diagnosis of Henoch- case? glomerulonephritis associated with Henoch- SchĂśnlein purpura?
The urine sediment in this case shows acanthocytes (a particular type of dysmorphic RBC) (see Figure 34-2A) and an RBC cast (see Figure 34-2B), which
Several types of GN can occur in association with infection, including HSP, PSGN, and IgA nephropathy.
The skin rash in this case (see Figure 34-1) is consistent with palpable purpura related to cutaneous small vessel vasculitis. This finding indicates systemic involvement and distinguishes HSP from IgA nephropathy.
Skin biopsy with light and immunofluorescence microscopy would reveal leukocytoclastic vasculitis and, more specifically, IgA deposition within the walls of the blood vessels, which is pathognomonic of HSP. Skin biopsy is less invasive than renal biopsy and is generally preferred. However, in cases of severe renal involvement, renal biopsy may have a role in providing prognostic information and guiding therapy. The principal finding of HSP on renal biopsy is globular mesangial IgA deposition on immunofluorescence (also seen with IgA nephropathy). 31 disease in adults compared with children. In patients withisevere GN, treatment options includetglucocorticoids either alone or in combination with plasma. Spontaneous remission of HSP and complete recovery of renal function occur in most cases, but here is a higher risk of progression to chronic kidney exchange or immunosuppressive agents such as azathiopr ne and intravenous immunoglobulins (IVIG). The efficacy of these modalities is largely unknown Ultimately, renal transplantation may be necessary for patients who progress to end-stage renal disease. 31
709 Key Points
Glomerular disease describes a heterogeneous group of entities characteristic clinical, physiologic, biochemical, and histologic that damage various glomerular structures and produce manifestations. Glomerular injury results in impaired filtration, allowing
constituents of blood into the urinary space that are normally excluded (eg, cells, protein).
The clinical manifestations of glomerular disease include nephrotic Nephrotic syndrome describes the constellation of peripheral hyperlipidemia. Renal excretory function can be variable. syndrome and nephritic syndrome and or the combination of both. edema, proteinuria >3.5 g/d, hypoalbuminemia, and Important sequelae of nephrotic syndrome include progressive renal failure, thromboembolism, and susceptibility to infection. y. constellation of hypertension and active urine sediment occasionally WBCs and WBC casts). Renal excretory function and patterns: positive ANCA, positive anti-GBM antibodies, low ic ANCA-associated GN can be associated with c-ANCA or p- Nephrotic syndrome can be primary (ie, idiopathic) or secondar Nephritic syndrome (ie, glomerulonephritis) describes the (dysmorphic RBCs [acanthocytes in particular], RBC casts, and degree of proteinuria can be variable. The causes of GN can be separated into the following serolog complement levels, and other. ANCA. Serologic patterns of GN are not 100% sensitive.
Renal biopsy is the single most definitive investigation into the cause of glomerular disease.
710
References 1. Longo DL, Fauci AS, Kasper DL, Hauser SL, Jameson JL, Loscalzo J, eds. Harrisonâs Principles of Internal Medicine. 18th ed. New York, NY: McGraw-Hill; 2012. Madaio MP, Harrington JT. The diagnosis of glomerular diseases: acute
- glomerulonephritis and the nephrotic syndrome. Arch Intern Med. 2001;161(1):25-34.
- Hull RP, Goldsmith DJ. Nephrotic syndrome in adults. BMJ. 2008;336(7654):1185-1189.
- Orth SR, Ritz E. The nephrotic syndrome. N Engl J Med. 1998;338(17):1202-1211.
-
Ginsberg JM, Chang BS, Matarese RA, Garella S. Use of single voided urine samples to estimate quantitative proteinuria. N Engl J Med. 1983;309(25):1543-1546. Waldman M, Crew RJ, Valeri A, et al. Adult minimal-change disease: clinical
-
characteristics, treatment, and outcomes. Clin J Am Soc Nephrol. 2007;2(3):445-453.
- pathogenesis, diagnosis, and treatment. J Formos Med Assoc. 2015;114(2):102-111. Lai WL, Yeh TH, Chen PM, et al. Membranous nephropathy: a review on the
-
Hogan J, Radhakrishnan J. The treatment of minimal change disease in adults. J Am Soc Nephrol. 2013;24(5):702-711.
-
Prakash J. Non-diabetic renal disease (NDRD) in patients with type 2 diabetes mellitus (type 2 DM). J Assoc Physicians India. 2013;61(3):194-199.
-
Wei Q, Zhang L, Liu X. Outcome of severe preeclampsia manifested as nephrotic syndrome. Arch Gynecol Obstet. 2011;283(2):201-204.
-
Christiansen CF, Onega T, Svaerke C, et al. Risk and-prognosis of cancer in patients 12. Kolasinski SL, Chung JB, Albert DA. What do we know about lupus membranous with nephrotic syndrome. Am J Med. 2014;127(9):871 877 e1. nephropathy? An analytic review. Arthritis Rheum. 2002;47(4):450-455.
-
Korbet SM, Schwartz MM. Multiple myeloma. J Am Soc Nephrol. 2006;17(9):2533- 2545.
-
Iskandar SS, Falk RJ, Jennette JC. Clinical and pathologic features of fibrillary glomerulonephritis. Kidney Int. 1992;42(6):1401-1407.
-
Kohler H, Wandel E, BrunckIB. Acanthocyturiaâa characteristic marker for 16Proposal of an international consensus.conference. Arthritis Rheum. 1994;37(2):187- glomerular bleeding. Kidney nt. 1991;40(1):115-120. . Jennette JC, Falk RJ, Andrassy K, et al Nomenclature of systemic vasculitides. 192. . Kubaisi B, Abu Samra K, Foster CS. Granulomatosis with polyangiitis (Wegenerâs
17disease): an updated review of ocular disease manifestations. Intractable Rare Dis Res. 2016;5(2):61-69.
-
Almouhawis HA, Leao JC, Fedele S, Porter SR. Wegenerâs granulomatosis: a.review of clinical features and an update in diagnosis and treatment. J Oral Pathol Med 2013;42(7):507-516.
-
Chung SA, Seo P. Microscopic polyangiitis. Rheum Dis Clin North Am. 2010;36(3):545-558.
-
Vaglio A, Buzio C, Zwerina J. Eosinophilic granulomatosis with polyangiitis (Churg- Strauss): state of the art. Allergy. 2013;68(3):261-273.
-
Troxell ML,JHoughton DC. Atypical anti-glomerular basement membrane disease. Clin Kidney . 2016;9(2):211-221.
- Ferretti JJ, Stevens DL, Fischetti VA, eds. Streptococcus pyogenes: Basic Biology to
711
Clinical Manifestations. Oklahoma City, OK; 2016. 23. Sethi S, Fervenza FC. Membranoproliferative glomerulonephritisâa new look at an old entity. N Engl J Med. 2012;366(12):1119-1131.
- Borchers AT, Leibushor N, Naguwa SM, Cheema GS, Shoenfeld Y, Gershwin ME. Lupus nephritis: a critical review. Autoimmun Rev. 2012;12(2):174-194. . Cameron JS, Vick RM, Ogg CS, Seymour WM, Chantler C, Turner DR. Plasma C3 an
25C4 concentrations in management of glomerulonephritis. Br Med J. 1973;3(5882):668- d 672.
-
Boils CL, Nasr SH, Walker PD, Couser WG, Larsen CP. Update on endocarditis- associated glomerulonephritis. Kidney Int. 2015;87(6):1241-1249.
-
Majumdar A, Chowdhary S, Ferreira MA, et al. Renal pathological findings in Infective endocarditis. Nephrol Dial Transplant. 2000;15(11):1782-1787.
-
DâAmico G. Renal involvement in hepatitis C infection: cryoglobulinemic glomerulonephritis. Kidney Int. 1998;54(2):650-671.
-
Ramos-Casals M, Stone JH, Cid MC, Bosch X. The cryoglobulinaemias. Lancet. 2012;379(9813):348-360.
-
Donadio JV, Grande JP. IgA nephropathy. N Engl J Med. 2002;347(10):738-748.
- Roberts PF, Waller TA, Brinker TM, Riffe IZ, Sayre JW, Bratton RL. Henoch-Schonlein purpura: a review article. South Med J. 2007;100(8):821-824.
712
CHAPTER 35
713
Hyperkalemia
714 Case: A 45-year-old woman with dark urine A 45-year-old woman with a history of type 1 diabetes mellitus and
coronary artery disease is brought to the emergency department after prescribed insulin therapy. Other medications include lisinopril and and respiratory rate is 30 breaths per minute. Breathing is deep and g, being found down by family. She has a history of poor adherence to metoprolol succinate. Heart rate is 124 beats per minute, blood pressure is 140/84 mm H labored. Serum sodium (Na ) is 130 mEq/L; potassium (K ), 6.6 mEq/L; + + chloride, 94 mEq/L; bicarbonate, 10 mEq/L; blood urea nitrogen, 48 mg/dL; creatinine, 2.1 mg/dL; and glucose, 800 mg/dL. Urine
sample is shown in Figure 35-1. No red blood cells are visualized on microscopic urinalysis.
FIGURE 35-1
What are the possible causes of hyperkalemia in this patient?
What is normal serum K+ concentration? What is pseudohyperkalemia? How is K+ normally distributed within the body? Why is it important to maintain normal extracellular K+ concentration? What processes are responsible for regulating serum K+?
What are the symptoms of hyperkalemia? What are the electrocardiographic manifestations of hyperkalemia?
The normal range for serum K+ may vary slightly between laboratories but is typically 3.6 to 5 mEq/L.1
Pseudohyperkalemia is the presence of elevated serum K+ in a blood sample due to efflux of K+ from red blood cells during the process of phlebotomy or after the sample has been drawn. Approximately 98% of total body K+ is sequestered within cells (mostly muscle cells), with the remaining 2% in the extracellular fluid compartment.1 The ratio of intracellular to extracellular K+ is the most important determining factor of the resting membrane potential of neurons and myocytes, which allows for the generation and propagation of action potentials necessary for normal function and stability of the heart and other muscles.2 The kidneys maintain total body K+ balance by matching excretion with intake, a hormonal process that occurs over a period of hours. The movement of K+ between the intracellular and extracellular compartments (ie, transcellular shift) regulates more acute changes in serum K . The Gastrointestinal tract normally clears around + 10% of K+ intake, a contribution that adaptively increases in the setting of renal failure.2,3 Symptoms of hyperkalemia may include muscle weakness, paresthesias, and palpitations.
Electrocardiographic manifestations of hyperkalemia include (in order of increasing severity) Increased amplitude and peaking of the T wave, prolonged PR interval, Decreased amplitude and eventual disappearance of the P wave, prolonged QRS interval, and sine wave pattern (Figure 35-2).4
715
What conduction abnormalities are associated with hyperkalemia? What cardiac dysrhythmias are associated with hyperkalemia? Which organ is chiefly responsible for maintaining total body K+ by matching elimination with intake?
What are the 2 subcategories of renal- associated hyperkalemia based on renal function?
What is the clinical surrogate for renal function? How is glomerular filtration rate estimated? What is normal serum creatinine concentration?
FIGURE 35-2 Electrocardiographic manifestations of hyperkalemia. (Adapted from Marini JJ, Wheeler AP. Critical Care Medicine: The Essentials. 4th ed. Philadelphia, PA: Lippincott Williams & Wilkins; 2010.)
Hyperkalemia can result in bundle branch block (right or left), bifascicular block, and Atrioventricular block.5
Hyperkalemia may be associated with sinus bradycardia, asystole, and idioventricular rhythms (eg, ventricular tachycardia, ventricular fibrillation).4
The kidneys are primarily responsible for maintaining total body K . +
Renal-associated hyperkalemia can occur in the setting of Decreased renal clearance or normal renal clearance.
Renal clearance can be estimated by calculating the glomerular filtration rate (GFR).
Several equations are used to predict GFR (most commonly the Modification of Diet in Renal Disease [MDRD] and the Cockcroft-Gault equations), each based primarily on serum creatinine concentration.6,7 Creatinine is produced and released by muscle without significant short-term (day-to-day) variation. Serum creatinine levels are therefore dependent on muscle mass, which is dependent on nutritional status, age, gender, and ethnicity. For the average adult man, the normal range for serum creatinine is 0.6 to 1.2 mg/dL; for the average adult woman, it is approximately 0.5 to 1.1 mg/dL. Normal serum creatinine values in individuals with muscle mass above or below average (eg, body builders, elderly, malnourished patients) may be outside of those ranges.8
716
Renal Causes of Hyperkalemia in the Setting of Decreased Renal Clearance
What is the biochemical definition of Acute Kidney Injury (AKI)? What is the definition of chronic kidney disease (CKD)? Is severe hyperkalemia more likely to occur with Acute Kidney Injury or chronic kidney disease?
AKI is defined by any of the following: (1) increase in serum creatinine by âĽ0.3 mg/dL within 48 hours, (2) increase in serum creatinine concentration to âĽ1.5 times baseline, known or presumed to have occurred within the prior 7 days, or (3) urine volume <0.5 mL/kg per hour for 6 hours.9
CKD is defined as the presence of kidney damage (eg, albuminuria) or Decreased kidney function (ie, GFR <60 mL/min/1.73 m2) for âĽ3 months.10
Severe hyperkalemia is more likely to occur in patients with acute rather than chronic kidney injury. Risk increases in parallel with the rate of renal function loss because there is less time for adaptation to occur. In the setting of chronic renal insufficiency, remaining nephrons develop an Increased ability to excrete K , an adaptive mechanism to maintain overall K+ balance. This mechanism is effective until GFR drops below 15 to 20 mL/min. Extrarenal + adaptations also occur, including Increased cellular uptake and Increased Gastrointestinal excretion of K .3,11 +
What are the causes of hyperkalemia related to Decreased renal clearance?
Avocados, spinach, sweet potatoes, oranges, and bananas. Think of this iatrogenic source of hyperkalemia in hospitalized patients. A 56-year-old woman with end-stage renal disease develops a trend of increasing serum K+ after being started on chronic narcotics for pain.
What natural beverage, readily served at Queenâs Park Savannah in Trinidad and Tobago, contains high K+ content?
Oral intake (including both dietary and iatrogenic K+ supplementation). Intravenous infusion of K . + Constipation.
High concentrations of K+ are found in coconut water. Dietary K+ restriction is an important part of the long-term management of patients with CKD. Patients should be thoroughly counseled regarding all possible sources of dietary K . +
What are the potential sources Sources of intravenous K+ in the hospital include K+ given to correct hypokalemia (patients with of intravenous K+? impaired renal function are at particular risk of overcorrection), potassium-containing medication (eg, penicillin G), and blood products. What factors increase the risk Risk of hyperkalemia rises with duration of red blood cell storage, irradiation of the blood, and large of hyperkalemia with blood transfusion volumes (even with fresh products).12 transfusion? What is the mechanism of Gastrointestinal excretion of K+ increases in patients with CKD, an adaptive mechanism designed to hyperkalemia related to maintain K+ homeostasis. When patients with CKD develop constipation, there is a reduction in overall constipation? K+ elimination.3 caExogenous intake of K , including via oral and intravenous1routes, is a rare + use of hyperkalemia in patients with normal renal function. 3
717
Renal Causes of Hyperkalemia in the Setting of Normal Renal Clearance
What are the renal causes of hyperkalemia in the setting of normal renal clearance?
A 56-year-old man develops Angiotensin-converting enzyme inhibitor (ACE-I). hyperkalemia after starting treatment for hypertension. Also known as renal tubular Hypoaldosteronism. acidosis type IV. Normal renal function, Pseudohypoaldosteronism. hyperkalemia, metabolic acidosis, and elevated serum aldosterone level.
What are the mechanisms of medication-induced hyperkalemia related to the kidney?
Mechanisms of medication-induced hyperkalemia related to the kidney include impaired aldosterone secretion (eg, ACE-I, nonsteroidal anti-inflammatory drugs) and impaired action of aldosterone (eg, aldosterone antagonists, potassium-sparing diuretics) (Figure 35-3).14
FIGURE 35-3 The renin-angiotensin-aldosterone system and regulation of renal K+ excretion. Aldosterone binds to a cytosolic receptor in the principal cell and stimulates Na+ reabsorption across the luminal membrane. As Na+ is reabsorbed, the electronegativity of the lumen increases thereby providing a more favorable driving force for K+ secretion. Disease states or drugs that interfere at any point along this process can impair renal K+ secretion and lead to hyperkalemia. (From Schrier RW. Diseases of the Kidney and Urinary Tract. 8th ed. Philadelphia, PA: Lippincott Williams & Wilkins; 2007.)
What are the causes of Causes of hypoaldosteronism include medications (eg, ACE-I), hyporeninemia, and primary adrenal hypoaldosteronism? insufficiency. What is Pseudohypoaldosteronism describes a group of disorders characterized by renal tubular unresponsiveness pseudohypoaldosteronism? to aldosterone. It results in hyperkalemia, metabolic acidosis, and Increased serum aldosterone Although theseccauses of hyperkalemia can occur in patients with normal oncentration. renal function, incidence and severity are generally amplified when superimposed on renal dysfunction.
718
719
Extrarenal Causes of Hyperkalemia
What is the principal mechanism of hyperkalemia unrelated to the kidney?
How is the concentration gradient of K+ between the intracellular and extracellular fluid compartments maintained? What are the chief factors that affect transcellular shifts of K+? What is the effect of insulin on the Na /K+ pump? + What are the effects of catecholamines on the Na /K+ + pump?
Hyperkalemia unrelated to the kidney occurs as a result of transcellular shift.
The concentration gradient of K+ between the intracellular and extracellular fluid compartments is maintained by the Na -K -adenosine triphosphatase (ie, Na /K -ATPase or Na /K+ pump), which uses energy to move K+ against its concentration gradient from the extracellular to the intracellular + + + + + compartment (Figure 35-4).2 Under normal conditions, insulin and catecholamines are the primary drivers of transcellular shifts of K . Acid-base derangements and plasma tonicity also affect transcellular shifts of K .2 + + Insulin increases the activity of the Na /K+ pump, thereby accelerating the movement of K+ into the + intracellular compartment. Insulin is often used in the treatment of hyperkalemia.2 β-Adrenergic receptors activate the Na /K+ pump, whereas ι-adrenergic receptors impair cellular + entry of K . β2-Agonists are often used in the treatment of hyperkalemia.2 +
FIGURE 35-4 Mechanisms regulating transcellular shifts in potassium. (From Porth CM. Essentials of Pathophysiology: Concepts of Altered Health States. 4th ed. Philadelphia, PA: Wolters Kluwer; 2015.)
What are the causes of hyperkalemia related to transcellular shift?
Hyperkalemia on induction of anesthesia. This condition can be identified with an arterial blood gas test. A metabolic abnormality common to hyperglycemia, mannitol, ethylene glycol, ethanol, and methanol toxicity. A cause of pseudohematuria, often associated with crush injuries. A 43-year-old woman develops hyperkalemia
Succinylcholine.
Acidemia.
Serum hyperosmolality.
Rhabdomyolysis.
Tumor lysis syndrome.
720
shortly after starting treatment for lymphoma. A result of pancreatic endocrine dysfunction. Episodes of transient weakness or paralysis beginning in infancy.
Insulin deficiency.
Hyperkalemic periodic paralysis.
Which medications Medications that lead to K+ movement out of cells include β-blockers, digoxin, and succinylcholine. promote movement of K+ to the extracellular compartment? What is the The increase in extracellular K+ related to acidemia results from the interplay between various ion channels. For mechanism of example, lower extracellular pH decreases the rate of Na -H+ exchange, which lowers intracellular Na , reducing the + + acidemia- associated activity of the Na /K+ pump and leading to higher extracellular K .2 + + transcellular shift of K+? What is the In the setting of hyperosmolality, K+ moves with water from the intracellular to the extracellular compartment mechanism of through the process of solvent drag. As water moves out of cells, intracellular K+ increases, producing a higher hyperkalemia associated with concentration gradient that favors the efflux of K .2 + hyperosmolality? What are the Rhabdomyolysis is associated with positive blood on urine dipstick (a result of myoglobinuria) in the absence of red classic urinalysis blood cells on microscopic evaluation. This combination of findings can be an important clue to the diagnosis. findings in the setting of rhabdomyolysis? What is the classic Tumor lysis syndrome describes a characteristic pattern of metabolic disturbances, including hyperkalemia, laboratory pattern hyperphosphatemia, hyperuricemia, and hypocalcemia, which develop as a result of the release of intracellular associated with content into the bloodstream when there is massive tumor cell lysis, either spontaneously or as a result of therapy. tumor lysis The clinical consequences can be severe, including renal failure, cardiac dysrhythmias, seizure, and death. In patients syndrome? at risk for tumor lysis syndrome, preventative strategies include intravenous hydration and the use of hypouricemic agents (eg, allopurinol, rasburicase).15 What are some Insulin deficiency can occur in the setting of diabetes mellitus (both types 1 and 2), fasting (particularly in dialysis scenarios in which patients), and treatment with somatostatin or somatostatin-agonists (particularly in dialysis patients).16,17 there may be a deficiency of insulin that leads to hyperkalemia? What are the Precipitants of weakness in patients with hyperkalemic periodic paralysis include cold exposure, rest after exercise precipitants of (often aborted by resuming exercise), hunger, low-carbohydrate meals, and potassium-rich meals.18,19 weakness in patients with hyperkalemic perAlthough these causes of hyperkalemia can occur in patients with normal iodic paralysis? renal function, incidence and severity are generally amplified when superimposed on renal dysfunction.
721
722 Case Summary coronary artery disease is brought to the hospitallafter being foundnd A 45-year-old woman with a history of type 1 diabetes mellitus a down and is discovered to have multiple metabo ic derangements, including hyperkalemia.
What are the possible causes of hyperkalemia in this patient? Rhabdomyolysis, Acute Kidney Injury, medication (ACE-I and β-blocker), insulin deficiency, and hyperosmolality.
723
this case? Rhabdomyolysis and associated myoglobinuria can result in red-brownâ, tea-, or cola-colored urine (see F gure 35-1). The absence of Bonus Questions
What is the significance of the urine sample in
What is the cause of low serum bicarbonate in What are the most likely causes of Acute Kidney this case? Injury in this case? What is thelcalculated serum osmolality in this + case? (Reca l serum Na is 130 mEq/L, BUN 48 mg/dL, and glucose 800 mg/dL.) What medication can be given to stabilize the myocytes in the setting of hyperkalemia? What medications can be used to temporarily shift K+ into the intracellular compartment? What medications can be used to increase elimination of K+ from the body? In addition to pharmacologic therapy, what therapeutic intervention may be necessary in cases of severe hyperkalemia?
RBCs onimicroscopic analysisrof the urine indicates that the color is not related to the presence of blood. Itiis not possible to differentiate The low serum bicarbonate in this case is the result of anion gap metabolic acidosis, likely caused by diabetic ketoacidosis. Diabetic ketoacidosis results in osmotic diuresis, which can lead to prerenal AKI. Rhabdomyolysis can also cause AKI via afferent hemoglob nuria and myoglobinu ia based on the appearance of urine alone. arteriole vasoconstriction and direct tubular toxicity. In this case, serum osmolality =(130 Ă2) +(48/2.8) +(800/18) =322 mOsm/kg. This is elevated (reference range 275-295 mOsm/kg) and could be contributing to the hyperkalemia.
An intravenous infusion of calcium chloride (10%) or calcium gluconate (10%) can be used to stabilize myocytes in the setting of hyperkalemia. Calcium gluconate is less irritating to the veins at the injection site. 2 13 β -Agonists (eg, albuterol), insulin (along with an ampule of dextrose in patients with normal serum glucose concentration), and sodium bicarbonate are often used to temporarily shift K+into the intracellular compartment (see Figure 35-4).13 Ion-exchange resins (eg, sodium polystyrene) can be used to increase Gastrointestinal excretion of K , and loop diuretics (eg, furosemide) can be used to increase renal excretion of K .13 + + In severe cases of hyperkalemia, renal replacement therapy (RRT) may ultimately be necessary to remove K+from the body. Hemodialysis is more effective than continuous forms of RRT.13
724 Key Points
exogenous intake, excretion in urine and stool, and transcellular Transcellular shift of K+ occurs rapidly and is primarily driven by Potassium homeostasis is regulated by the balance between shift. insulin and catecholamines. + Maintenance of normal serum K levels is important for a var of reasons, including stability of the heart and other muscles.iety Hyperkalemia is generally defined as serum K >5 mEq/L. Clinical manifestations of hyperkalemia include weakness, +
paresthesias, palpitations, electrocardiographic changes, and dysrhythmia.
The kidney is the principal organ responsible for regulating total body K . Hyperkalemia can be caused by renal or extrarenal processes. +
Renal causes of hyperkalemia can be separated according to renal In patients with impaired renal function, the risk of hyperkalemia function. is generally higher with AKI, as adaptive mechanisms occur in the setting of CKD to maintain K homeostasis. + Extrarenal causes.of hyperkalemia are primarily related to transcellular shift The etiologies of hyperkalemia in the setting of normal renal
function also occur in the setting of renal dysfunction, generally Management of hyperkalemia depends on its severity and rate of Intravenous calcium can be used to stabilize the myocytes in cells (eg, albuterol) and increase K texcretion in urine (eg, into Renal replacement therapy may ultimately be necessary to treat with more severe results. development. hyperkalemic patients. Pharmacologic agents can be used o promote influx of K + + furosemide) and stool (eg, sodium polystyrene). hyperkalemia, particularly in severe cases.
725
References 1. AronsoniPS, Giebisch G. Effects of pH on potassium: new explanations for old 2. Palmer BF. Regulation of potassium homeostasis. Clin J Am Soc Nephrol. observat ons. J Am Soc Nephrol. 2011;22(11):1981-1989. 2015;10(6):1050-1060.
- Hayes CP Jr, McLeod ME, Robinson RR. An extrarenal mechanism for the maintenance of potassium balance in severe chronic renal failure. Trans Assoc Am Physicians.
1967;80:207-216. 4. Mattu A, Brady WJ, Robinson DA. Electrocardiographic manifestations of hyperkalemia. Am J Emerg Med. 2000;18(6):721-729.
-
Bashour T, Hsu I, Gorfinkel HJ, Wickramesekaran R, Rios JC. Atrioventricular and intraventricular conduction in hyperkalemia. Am J Cardiol. 1975;35(2):199-203.
-
Cockcroft DW,;Gault MH..Prediction of creatinine clearance from serum creatinine. Nephron. 1976 16(1):31-41 Levey AS, Bosch JP, Lewis JB, Greene T Rogers N, Roth D. A more accurate method t
-
estimate glomerular filtration rate from,serum creatinine: a new prediction equation.-o Modification of Diet in Renal Disease Study Group. Ann Intern Med. 1999;130(6):461 470.
-
WalkertHK, Hall WD, Hurst JW, eds. Clinical Methods: The History, Physical, and Labora ory Examinations. 3rd ed. Boston: Butterworths; 1990.
-
KDIGO AKI Work Group. KDIGO clinical practice guidelines for Acute Kidney Injury. Kidney Int Suppl. 2012;17:1-138.
-
KDIGO. Chapter 1: Definition and classification of CKD. Kidney Int Suppl. 2013;3:19. 11adaptation in,potassium excretion associated with nephron reduction in the dog. J Clin . Schultze RG Taggart DD, Shapiro H, Pennell JP, Caglar S, Bricker NS. On the Invest. 1971;50(5):1061-1068.
-
Vraets(A, Lin Y, Callum JL. Transfusion-associated hyperkalemia. Transfus Med Rev. 2011;25 3):184-196.
-
Lehnhardt A, Kemper MJ. Pathogenesis, diagnosis and management of hyperkalemia. Pediatr Nephrol. 2011;26(3):377-384.
-
Ben Salem C, Badreddine A, Fathallah N, Slim R, Hmouda H. Drug-induced hyperkalemia. Drug Saf. 2014;37(9):677-692.
-
Howard SC, Jones DP, Pui CH. The tumor lysis syndrome. N Engl J Med. 2011;364(19):1844-1854.
-
Adabala M, Jhaveri KD, Gitman M. Severe hyperkalaemia resulting from octreotide use in a haemodialysis patient. Nephrol Dial Transplant. 2010;25(10):3439-3442.
-
Allon M, Takeshian A, Shanklin N. Effect of insulin-plus-glucose infusion with or without epinephrine on fasting hyperkalemia. Kidney Int. 1993;43(1):212-217.
-
Fontaine B, Lapie P, Plassart E,:et al. Periodic paralysis and voltage-gated ion channels. Kidney Int. 1996;49(1) 9-18.
- MilleriTM, Dias da Silva MR, Miller HA, et al. Correlating phenotype and genotype in the per odic paralyses. Neurology. 2004;63(9):1647-1655.
726
727
CHAPTER 36
728
Hypernatremia
729 Case: A 28-year-old woman with polyuria to the emergency department afterfdeveloping confusion. The patientt making frequent trips to urinate. She acknowledged at that time that A 28-year-old woman with no known medical conditions is brough was camping in eastern Oregon be ore symptoms began. During the trip, friends noticed that she was unusually thirsty and had been she had noticed these symptoms for several months but had not sought medical evaluation. On the third day, after the water supply
started.to diminish, the patient reported to friends that she was feeling unwell She was taken to the emergency room when she began saying things that did not make sense. Heart rate is 75 beats per minute, and blood pressure is
123/84 mm Hg. The patient is somnolent and not oriented. Mucous membranes and axillae are moist. Jugular venous pressure is
estimated to be 6 cm H2O. There is no peripheral edema. + Serum sodium (Na ) is 158 mEq/L, blood urea nitrogen (BUN) is 12 mg/dL, creatinine is 0.7 mg/dL,/and osmolalityiis 326 mOsm/kg What is the most likely cause of hypernatremia in this patient? + (reference range 275 to 295 mOsm kg). Urine Na s 70 mEq/L and osmolality is 152 mOsm/kg.
What is hypernatremia? What is normal serum Na+ concentration? How is Na+ normally distributed within the body? How is water homeostasis regulated?
Hypernatremia is defined as an elevated serum Na+ concentration that occurs when there is a deficit of water relative to Na+ in the extracellular fluid compartment.1 Normal serum Na+ is 135 to 142 mEq/L.2
Most Na+ resides in extracellular fluid, and its distribution between extracellular and intracellular fluid is regulated by the Na /K -adenosine triphosphatase (ie, Na /K -ATPase or Na /K+ pump).3 + + + + +
The central nervous system and the kidneys work in concert to maintain water homeostasis, which ultimately controls serum Na+ concentration. Osmoreceptors located in the hypothalamus detect extracellular tonicity and respond by adjusting thirst and the secretion of vasopressin (ie, arginine vasopressin [AVP], or antidiuretic hormone [ADH]). Vasopressin acts in the kidney to increase the reabsorption of free water; it can concentrate urine to a maximum of 1200 mOsm/kg. In the absence of vasopressin, urine osmolality can drop to a minimum of 50 mOsm/kg. In the setting of hypernatremia, thirst and vasopressin secretion should be stimulated. A significant decrease in blood volume also stimulates vasopressin secretion (Figure 36-1).2
FIGURE 36-1 When osmoreceptors in the hypothalamus sense an increase in solute concentration, or when baroreceptors in the hypothalamus sense a decrease in blood volume, there is an increase in the sensation of thirst and the secretion of
730
What are the effects of serum Na+ concentration on cell volume?
vasopressin (ADH) from the posterior pituitary, which leads to water reabsorption in the distal tubules of the kidneys. Homeostasis is maintained by a negative feedback loop. (Modified from Porth CM. Essentials of Pathophysiology: Concepts of Altered Health States. Philadelphia, PA: Lippincott Williams & Wilkins; 2003, with permission.)
Sodium is an effective solute and therefore contributes to overall serum tonicity. Water moves freely between fluid compartments in response to differences in tonicity. Accordingly, the presence of hypernatremia, which is always associated with hypertonicity, causes a shift of water out of cells, resulting in cellular contraction. Conversely, hypotonic hyponatremia causes a shift of water into cells, resulting in cellular swelling (Figure 36-2).3
FIGURE 36-2 The effects of hypernatremia and hyponatremia on cell volume and extracellular fluid (ECF). (From Smeltzer SC, Hinkle JL, Bare BG, Cheever KH. Brunner and Suddarthâs Textbook of Medical-Surgical Nursing. 12th ed. Philadelphia, PA: Wolters Kluwer; 2010.)
What are the In most tissues of the body, the Na+ concentrations of serum and interstitial fluid are virtually identical due to free effects of movement of Na+ through the capillary membrane. In contrast, capillaries in the brain are impermeable to Na . The + serum Na+ result is that an abnormal serum Na+ concentration will cause water to move into or out of brain tissue, with subsequent concentration on brain tissue? swelling or contraction, respectively. Only minimal changes in the volume of brain tissue are compatible with life.2 What adaptive In the setting of chronic hypernatremia, defined by at least 24 to 48 hoursâ duration, osmotically active intracellular mechanisms molecules (osmolytes) (eg, glutamate, taurine, myo-inositol) are Increased within cells, including brain cells. This helps occur in the decrease the gradient of tonicity between extracellular and intracellular fluid, which in turn, decreases movement of setting of water out of cells. This adaptation is particularly important in brain tissue.2 chronic hypernatremia? What are the Given the adaptation that occurs in response to chronic hypernatremia, patients are often asymptomatic. In contrast, clinical patients with acute hypernatremia (developed over a period of hours) are almost always symptomatic, and changes in differences brain volume can be life-threatening. The physiologic adaptation that occurs in chronic hypernatremia limits the rate between acute with which hypernatremia can be corrected (Figure 36-3). In cases where duration is unknown, chronicity should be and chronic 2 assumed. hypernatremia?
FIGURE 36-3 Rapid onset or rapid correction of either hyponatremia or hypernatremia can cause brain damage. A rapid
What are the clinical manifestations of acute hypernatremia? What is the first step in
increase in the level of serum sodium, either from acute hypernatremia or from rapid correction of chronic hyponatremia, can cause osmotic demyelination (which can involve pontine and extrapontine regions of the brain). Cerebral edema is a complication of rapid decreases in serum sodium concentration, which can lead to brain herniation in severe cases. (Adapted with permission from Sterns RH. Disorders of plasma sodiumâcauses, consequences, and correction. N Engl J Med. 2015;372(1):55-65.)
Clinical manifestations of acute hypernatremia may include thirst (with some exceptions), muscle weakness, Decreased consciousness, delirium, convulsions, coma, and brain shrinkage, which can lead to osmotic demyelination, and vascular rupture with intracranial hemorrhage.2
Determining extracellular fluid volume status is the first step in establishing the cause of hypernatremia.
731
establishing the cause of hypernatremia?
What physical findings are associated with hypovolemia? What physical findings are associated with hypervolemia?
Hypovolemia may be associated with Decreased skin turgor, dry mucous membranes and axillae, Increased capillary refill time, sunken eyes, and low jugular venous pressure.
Hypervolemia may be associated with elevated jugular venous pressure, ascites, rales on lung auscultation (due to pulmonary edema), dullness to percussion of the lung bases (due to pleural effusions), and peripheral edema.
732
Hypovolemic Hypernatremia
Hypovolemic hypernatremia can be thought of as extracellular loss of hypotonic fluid (water > salt).
What are the 2 subcategories of hypovolemic hypernatremia?
What laboratory test can be helpful in distinguishing renal from extrarenal causes of hypovolemic hypernatremia?
Hypovolemic hypernatremia can be caused by renal or extrarenal processes.
Spot urine Na+ concentration can be suggestive of whether the source of volume depletion is renal (UNa >20 mEq/L) or extrarenal (UNa <20 mEq/L, especially <10 mEq/L).4
What are the renal causes of hypovolemic hypernatremia?
Iatrogenic loss of hypotonic fluid (relative to serum). Water follows the solute load. Opening the floodgates.
What is the mechanism of hypernatremia related to loop diuretics?
Diuretic medication, particularly loop diuretics.
Osmotic diuresis. Postobstructive diuresis.
Loop diuretics decrease the reabsorption of Na+ and chloride in the thick ascending limb of the loop of Henle (within the renal medulla), which ultimately impairs the ability of the kidneys to make concentrated urine (Figure 36-4). If the osmolality of excreted urine is lower than that of serum, then hypernatremia will eventually occur. Patients with diuretic- induced hypernatremia may be euvolemic or hypervolemic.5
FIGURE 36-4 Diuretic drugs are secreted into the proximal convoluted tubule and act at the sites shown. Approximately 70% of
Which particular type
filtered sodium is reabsorbed in the proximal convoluted tubule; 25%, in the thick ascending limb of the loop of Henle; 5%, in the distal convoluted tubule; and 1% to 2%, in the cortical collecting tubule (mediated by the action of aldosterone). Vasopressin (ADH) increases the permeability of the distal nephron for water. (Adapted with permission from Lilly LS. Pathophysiology of Heart Disease:lA Collaborative Project of Medical Students and Faculty. 6th ed. Philadelphia, PA: Wolters K uwer Health; 2016.)
Hyperosmolality caused by either effective solutes (eg, glucose) or ineffective solutes (eg, urea) can lead to the excretion of hypotonic urine through osmotic diuresis. Under certain circumstances, ineffective solutes can become effective urine solutes.
733
of osmo ic
diuresistis associated with hypernatremia? What is postobstructive diuresis?
However, only osmotic diuresis caused by ineffective solutes, such as urea, can result in hypernatremia. In the case of osmotic diuresis caused by effective solutes, such as glucose, the Increased tonicity of extracellular fluid triggers the movement of water from the intracellular to the extracellular fluid compartment, resulting in dilutional hyponatremia, which is the dominant effect. Postobstructive diuresis refers to a state of polyuria that occurs after relief of urinary tract obstruction, such as from bladder outlet obstruction or bilateral ureteral obstruction. Patients are at risk for dehydration, electrolyte disturbances (eg, hypernatremia), and death.6
What are the extrarenal causes of hypovolemic hypernatremia?
A patient develops hypernatremia after treatment for small bowel obstruction is initiated. Marathon runners and burn victims.
What are some causes of hypotonic fluid loss from the Gastrointestinal (GI) tract? What is the concentration of Na+ in sweat?
Loss of hypotonic fluid (relative to serum) related to nasogastric tube drainage.
Cutaneous (integumentary) loss of hypotonic fluid (relative to serum).
Loss of hypotonic fluid (relative to serum) from the GI tract can occur as a result of diarrhea, vomiting, or tube drainage. The average concentration of Na+ in sweat is approximately 40 mEq/L, but there is variation according to the site of the body and the rate of sweat production. Sweat Na+ concentration rises with Increased rates of sweat production but is always lower than serum Na+ concentration.7
734
Euvolemic Hypernatremia Euvolemic hypernatremia can be thought of as extracellular deficiency of pure water.
What are the causes of euvolemic hypernatremia?
Hypernatremia develops despite minimal loss of free water in the kidneys, GI tract, or skin. Hypernatremia and polyuria. Transient hypernatremia (resolving within minutes).
What are the risk factors for hypernatremia due to low free water intake? What are the 2 general types of diabetes insipidus?
What are the causes of intracellular water shift?
Low free water intake.
Diabetes insipidus (DI). Intracellular water shift.
Risk factors for hypernatremia from low free water intake include older age, delirium, and intubation.1
DI can be central (ie, deficient vasopressin secretion) or nephrogenic (ie, vasopressin resistance). In the setting of hypernatremia, low urine osmolality (<300 mOsm/kg) is suggestive of DI. Unlike nephrogenic DI, central DI is responsive to treatment with desmopressin.8 An increase in intracellular osmolality, which is accompanied by the movement of water from the extracellular to the intracellular fluid compartment, can be caused by intensive exercise, and seizure from electroconvulsive therapy.9,10
735
Hypervolemic Hypernatremia Hypervolemic hypernatremia can be thought of as extracellular gain of hypertonic fluid (salt >water).
What are the causes of hypervolemic hypernatremia?
Look for concurrent hypertension. A castaway should never drink seawater.
What are the causes of primary hyperaldosteronism?
What are the sources of exogenous Na+ intake?
Mineralocorticoid excess. Exogenous Na+ intake.
Primary hyperaldosteronism (ie, Connâs syndrome) refers to a group of disorders in which production of aldosterone is excessively high relative to serum Na+ concentration. Manifestations may include hypertension, mild hypernatremia, and hypokalemia. Causes of primary hyperaldosteronism include adrenal hyperplasia, adrenal adenoma, adrenocortical carcinoma, and familial hyperaldosteronism.11 Sources of exogenous Na+ intake include hypertonic sodium bicarbonate infusion, hypertonic sodium chloride infusion, hypertonic tube feeds, ingestion of sodium chloride, ingestion of seawater, emetics rich in sodium chloride, hypertonic saline enemas, hypertonic dialysis, and intrauterine injection of hypertonic saline.1
736 Case Summary A 28-year-old woman with chronic thirst and polyuria presents with
acute confusion and somnolence, and is found to have hypernatremia. What is the most likely cause of hypernatremia in this patient? Diabetes insipidus.
737
Bonus Questions
Which general This case can be described as euvolemic hypernatremia. category of hypernatremia
applies to this case? What features The normal jugular venous pressure and absence of peripheral edema in this case effect vely rule out hypervolemia. Euvolemia and hypovolemia can be difficult to in this case are differentiate clinically. However, the presence of moist mucous membranes and axillae,ilow BUN, and high urine Na+concentration suggest that extracellular fluid volume is not low.12 suggestive of euvolemic volume status? What iss? DI is a syndromelcharacterized by the production of excessive volumes (>50 mL/kg body weightiper 24ihours) of dilute urine (osmolality <300 mOsm/kg). Symptoms insipidu diabetes include thirst, po yuria, enuresis, and nocturia. Dehydration does not usually occur unless fluid ntake s impaired. 13 What is central Central DI is characterized by insufficient secretion of vasopressin in the central nervous system, which leads to hypotonic polyuria and hypernatremia. It can bes (eg, diabetes inherited or acquired. Common associations in adults include head trauma, neurosurgery, autoimmune destruction of the neurohypophysis, infiltrative disorder insipidus? sarcoidosis), malignancy, and ischemia. A significant proportion of cases are idiopathic. 8 What isenic Nephrogenic DI is characterized by renal resistance to vasopressin, which leadslto hypotonic polyuria and hypernatremia. It can be inherited or acquired. Common diabetes nephrog associations in adults include medications (eg, lithium), hypercalcemia, hypoka emia, and infiltrative disorders (eg, amyloidosis). 8 insipidus? Does then this Based on the information given, it islnot clear whether the patient in this case has central DI or nephrogenic DI. Testing would be needed to first confirm the diagnosis case have central or patient i of DI, and then further testing, to de ineate between central and nephrogenic forms.
nephrogenic diabetes insipidus? diagnosis of (eg, serum and urine osmolality) until 1 ofi2 endpoints is reached. If urine osmolality does not rise appropriately (>300 mOsm/kg) before serum osmolality (or/serums confirmed in complete DI is not present, andtother causes of polyuria should be considered, including partial DI andiprimary polydipsia (a higher threshold of urine osmolalityican laboratory test a synthetictvasopressin analogue. In patients with central DI, a full response totdesmopressin would be expected (urine volume will decline while urine osmolality ssin, How can the The water deprivation test can be used to confirm the diagnosis of DI. The patient is restricted from water and monitored closely for changes in laboratory parameter insipidus be + diabetes Na concentration) rises above the upper l mit of normal (295 mOsm/kg), then a diagnosis of complete DI is made. If urine osmolality does rise to >300 mOsm kg, then this case? be used to rule out partial DI, eg, 600 mOsm/kg). Assays have recently been developed to diagnose DI, ncluding measurement of serum copeptin, which may obv ate the need for water deprivation esting. 8,13,14 What Following he water deprivation test, when serum osmolality is sufficiently high and urine osmolality is inappropriately low, the patient should be given desmopre differentiate central from can be used to rises). In patients with nephrogenic DI, because of resistance to desmopressin, here would be no appreciable changes to urine volume and osmolality. 13
nephrogenic diabetes insipidus? Is thenatremia Given that the patient in this case is symptomatic, acute hypernatremia is most likely. The recent history of water deprivation during the camping trip is also indicative in this case acute or hyper of an acute process.
chronic? significance of patients with DI are able to achieve relatively normal serum Na tconcentrations, assuming there is adequate access to water. In this case, a lack of adequate water access led to an abrupt rise in serum Na . What is the Given the history of polyuria and Increased thirst, it is likely tha the patient in this case has had DI for at least several months. With an intact thirst mechanism, most + the history of polyuria and + Increased thirst in this case? What is thet of For symptomatic acute hypernatremia (<24-48 hours), rapidttreatment is imperative. This can be achieved by infusing a hypotonic fluid (eg, 5% dextrose in water) with edema and seizure. Correction of serum Na+at a rate of <0.5 mEq/L per hour (<12 mEq/d) is recommended in cases of chronic hypernatremia.2 hypernatremia? + managemen the aim of immediately restoring normal serum Na concen ration. Chronic hypernatremia should be corrected more cautiously to prevent events such as cerebral
738 Key Points
+ Water homeostasis, which controls serum Na concentration, is regulated by thirst and the hormonal interplay between the central nervous system and the kidneys.
Maintenance of normal serum Na+ concentration is important for preserving cell volume. Hypernatremia is generally defined as serum Na >142 mEq/L.
+ Clinical manifestations of hypernatremia depend on its duration and severity. Acute hypernatremia is often symptomatic with manifestations
that include thirst, muscle weakness, Decreased consciousness, to osmotic demyelination, and vascular rupture with intracranial delirium, convulsions, coma, and brain shrinkage, which can lead hemorrhage. Chronic hypernatremia is often asymptomatic.
Hypernatremia can be associated with extracellular hypovolemia, Hypovolemic hypernatremia results from extracellular loss of Hypovolemic hypernatremia can be caused by renal or extrarenal Euvolemic hypernatremia results from extracellular deficiency of Hypervolemic hypernatremia.results from extracellular gain of euvolemia, or hypervolemia. hypotonic fluid (water >salt). processes. pure water. hypertonic fluid (salt >water) Acute symptomatic hypernatremia should be corrected rapidly with infusion of hypotonic fluid and serial monitoring of serum Na concentration. + Chronic hypernatremia should be corrected judiciously to prevent cerebral edema and seizures.
739
References 1. Adrogue HJ, Madias NE. Hypernatremia. N Engl J Med. 2000;342(20):1493-1499. 2. Sterns RH. Disorders of plasma sodiumâcauses, consequences, and correction. N Engl J Med. 2015;372(1):55-65. Spital A, Sterns RD The paradox of sodiumâs volume of distribution. Why an
-
extracellular solute.appears to distribute over total body water. Arch Intern Med. 1989;149(6):1255-1257.
-
Liamis G, Filippatos TD, Elisaf MS. Evaluation and treatment-of hypernatremia: a practical guide for physicians. Postgrad Med. 2016;128(3):299 306. Szatalowicz VL, Miller PD, Lacher JW, Gordon JA, Schrier RW. Comparative effect o
-
diuretics on renal water excretion in hyponatraemic oedematous disorders. Clin Sci f (Lond). 1982;62(2):235-238.
-
Halbgewachs C, Domes T.iPostobstructive diuresis: pay close attention to urinary retention. Can Fam Physic an. 2015;61(2):137-142.
-
Schwartz IL, Thaysen JH. Excretion of sodium and potassium in human sweat. J Clin Invest. 1956;35(1):114-120.
-
Fenske W, Allolio B. Clinical review: current state and future perspectives in the diagnosis of diabetes insipidus: a clinical review. J Clin Endocrinol Metab.
2012;97(10):3426-3437. 9. Felig P, Johnson C, Levitt M, Cunningham;J, Keefe F, Boglioli B. Hypernatremia induced by maximal exercise. JAMA. 1982 248(10):1209-1211. 10. Welt LG, Orloff;J, Kydd DM, Oltman JE. An example of cellular hyperosmolarity. J 11case detection, diagnosis, and treatment: an Endocrine Society Clinical Practice nism: Clin Invest. 1950 29(7):935-939. . Funder JW, Carey RM, Mantero F, et al. The management of primary aldostero Guideline. J Clin Endocrinol Metab. 2016;101(5):1889-1916.
-
Ellison DH, Berl T. Clinical practice. The syndrome of inappropriate antidiuresis. N Engl J Med. 2007;356(20):2064-2072.
-
KalrarS, Zargar AH, Jain SM, et al. Diabetes insipidus: the other diabetes. Indian J 14diagnosis of the polyuria-polydipsia syndrome: a Prospective Multicenter Study. J Clin Endoc inol Metab. 2016;20(1):9-21. . Timper K, Fenske W, Kuhn F, et al. Diagnostic accuracy of copeptin in the differential Endocrinol Metab. 2015;100(6):2268-2274.
740
CHAPTER 37
741
Hypokalemia
742 Case: A 43-year-old woman with dry mouth A 43-year-old woman presents to the clinic with several weeks of that have been increasing in frequency. Before these symptoms began, difficulty swallowing, and vaginal dryness. Recent medical history is diBlood pressure is 118/82 mm Hg, and respiratory rate is 22 breaths filter paper is placed inside the lower eyelids, and the patient is asked paper strip from the right eye and 3.1 mm of moisture on the paper generalized weakness, muscle aches, and episodes of muscle cramps she reports a long-standing history of dry eyes, dry mouth with notable for spontaneous passage of a kidney stone. She denies arrhea. She does not take any medications. per minute. There is bilateral conjunctival erythema. A small strip of to close her eyes. After 5 minutes, there is 2.8 mm of moisture on the strip from the left eye (normal wetting is âĽ5 mm). Serum sodium (Na+) is 138 mEq/L; potassium (K+), 1.8 mEq/L; chloride (Cl), 116 mEq/L; bicarbonate (), 12 mEq/L; blood urea - nitrogen, 21 mg/dL; creatinine, 1.1 mg/dL; and glucose, 102 mg/dL. , Serum pH is 7.30. Urine sodium is 30 mEq/L, potassium is 40 mEq/L and chloride is 38 mEq/L. Urine pH is 6.80.
Electrocardiogram is shown in Figure 37-1.
FIGURE 37-1 (Courtesy of Ignatius Zarraga, MD.)
What is the most likely cause of hypokalemia in this patient?
What is normal serum K+ concentration? How common is hypokalemia? How is K+ normally distributed within the body? Why is it important to maintain a normal extracellular K+ concentration? What systems are responsible for regulating serum K+ concentration? What are the symptoms of hypokalemia?
The normal range for serum K+ may vary slightly between laboratories, but is typically 3.6 to 5 mEq/L.1
Hypokalemia is one of the most common electrolyte abnormalities, affecting around one-fifth of hospitalized patients.2 Approximately 98% of total body K+ is sequestered within cells (mostly muscle cells), with the remaining 2% in the extracellular fluid compartment.1 The ratio of intracellular to extracellular K+ is the most important determining factor of the resting membrane potential of neurons and myocytes, which allows for the generation and propagation of action potentials necessary for normal function and stability of the heart and other muscles.3 The kidneys maintain total body K+ balance by matching excretion with intake, a hormonal process that occurs over a period of hours. The movement of K+ between the intracellular and extracellular compartments (ie, transcellular shift) regulates more acute changes in serum K+ concentration. The Gastrointestinal (GI) tract normally clears around 10% of K+ intake.3 Symptoms of hypokalemia may include malaise, muscle weakness, myalgias, cramping, and constipation.4
743
electrocardiographic manifestations of What are the hypokalemia?
Which cardiac dysrhythmias are associated with hyperkalemia? What are the 3 general mechanisms of hypokalemia?
Electrocardiographic manifestations of hypokalemia include (in order of increasing severity) Decreased amplitude and broadening of the T wave, depression of the ST segment, increase in P-wave amplitude and duration, increase in QRS duration, the emergence of the U wave (most commonly seen in precordial leads V2 and V3), and fusion of the T and U waves (Figure 37-2).2
FIGURE 37-2 ECG manifestations of hypokalemia. (From Marino PL. Marinoâs The ICU Book. 4th ed. Philadelphia, PA: Wolters Kluwer Health/Lippincott Williams & Wilkins; 2014.)
Hypokalemia can result in tachyarrhythmia (including ventricular tachycardia and ventricular fibrillation) and Atrioventricular block.2
Hypokalemia can be caused by low intake, excess loss, or transcellular shift of K . +
744
+
Hypokalemia Related to Low Oral Intake of Potassium
When K+ intake is zero, what happens to total body K+?
Despite the presence of compensatory mechanisms to maintain total body K , when intake is zero, obligatory K+ losses in urine and stool still occur, resulting in the net loss of K+ over time.5
What are the causes of hypokalemia related to poor intake?
A 48-year-old man with a history of chronic heavy alcohol Poor oral intake. consumption presents with weakness and is found to have hypophosphatemia and hypokalemia.
What is the average daily oral K+ intake in the The average oral K+ intake in the industrialized world is approximately industrialized world? 75 mEq/d for men and 55 mEq/d for women.5 Under normal circumstances, what proportion of dietary K+ Approximately 85% to 90% of dietary K+ is absorbed in the GI tract; the vast is absorbed in the Gastrointestinal tract? majority of which occurs in the small intestine.5 What compensatory mechanism preserves K+ homeostasis When oral intake of K+ is low, there is a compensatory reduction in renal when oral intake is low? and GI excretion of K . When dietary K+ intake is zero, obligatory renal + losses decrease to approximately 5 to 10 mEq/d.5
745
Hypokalemia Related to Excess Loss of Potassium
What is the main source of K+ loss from the body?
The kidneys are the main source of K+ excretion (Figure 37-3).
FIGURE 37-3 K+ excretion depends on 2 factors. First, the hormone aldosterone must be present. Aldosterone increases the activity of the basolateral Na+/K+- ATPase, which generates the gradient for Na+ movement from the tubule into the cell. Second, there must be adequate distal delivery of Na+ to supply the epithelial Na+ channel. Intracellular movement of Na+ generates a negative charge within the tubular lumen, which is the driving force for K+ excretion via apical channels. (From Danziger J, Zeidel M, Parker MJ, Schwartzstein RM. Renal Physiology: A Clinical Approach. Philadelphia, PA: Lippincott Williams & Wilkins; 2012.)
746
Hypokalemia Related to Renal Loss of Potassium
What are the causes of hypokalemia related to renal loss of potassium?
Iatrogenic. Another electrolyte is to blame. Polyuria in a patient with hyperglycemia. Hypertension and hypokalemia. Increased intraluminal negative charge promotes K+ excretion in the kidney. A cause of non-anion gap metabolic acidosis. This genetic condition mimics the mechanism of thiazide diuretics. This genetic condition mimics the mechanism of loop diuretics.
Which medications are associated with renal K+ loss? Why is it important to evaluate for hypomagnesemia in patients with hypokalemia? What is the mechanism of hypokalemia in patients with osmotic diuresis?
Medication. Hypomagnesemia. Osmotic diuresis. Mineralocorticoid excess. Nonreabsorbable anions.
Renal tubular acidosis (RTA). Gitelmanâs syndrome. Bartterâs syndrome.
Medications commonly associated with hypokalemia include loop diuretics, thiazide diuretics, antimicrobials (eg, amphotericin B), mineralocorticoids (eg, fludrocortisone), and glucocorticoids (eg, prednisone).4 Coexistent hypomagnesemia can impair K+ repletion. It must be corrected for K+ supplementation to be completely effective.6 Osmotic diuresis results in Increased Na+ and water delivery to the distal tubule, which promotes Na+ and K+ exchange (see Figure 37-3).
What are the causes of aldosterone-independent syndromes of mineralocorticoid excess? What are some examples of nonreabsorbable anions?
Which types of renal tubular acidosis are associated with hypokalemia? Which syndrome is more common, Bartterâs or Gitelmanâs?
Aldosterone-independent causes of mineralocorticoid excess include Cushingâs syndrome, congenital adrenal hyperplasia, apparent mineralocorticoid excess, licorice ingestion, glucocorticoid resistance, exogenous mineralocorticoids, Liddle syndrome, and Geller syndrome. Examples of nonreabsorbable anions include bicarbonate in patients with vomiting, β-hydroxybutyrate in diabetic ketoacidosis, and penicillin antibiotics. These anions increase the intraluminal negative charge of the distal nephron, promoting K+ excretion.7 Non-anion gap metabolic acidosis is a consequence of all types of RTA. Types 1 and 2 in particular are associated with hypokalemia. RTA type 4 (ie, hypoaldosteronism) is associated with hyperkalemia.8 Gitelmanâs syndrome is significantly more prevalent in the general population (1 in 40,000) compared with Bartterâs syndrome (1 in 1,000,000).9
747
Hypokalemia Related to Extrarenal Loss of Potassium
What are the causes of hypokalemia related to extrarenal loss of potassium?
The most common extrarenal source of K+ loss from the body. Hypokalemia in marathon runners. Iatrogenic losses.
Gastrointestinal losses (eg, diarrhea, vomiting, tube drainage).
Excessive perspiration. Dialysis and plasmapheresis.
What is average fecal K+ excretion In the industrialized world, normal fecal K+ excretion averages 9 mEq/d. Excretion >16 to 22 mEq/d is per day? excessive and may cause hypokalemia, particularly, if it occurs over a prolonged period of time.5 What is the concentration of K+ in The average concentration of K+ in sweat is approximately 9 mEq/L. Individuals who exercise in hot sweat? climates are capable of secreting >12 L of sweat per day.10 How common is hypokalemia in Hypokalemia affects up to one-third of patients on peritoneal dialysis. It tends to occur more often in peritoneal dialysis patients? older patients and those with diabetes mellitus.11 What is the mechanism of Hypokalemia associated with plasmapheresis occurs as a result of dilution when plasma is removed hypokalemia related to and replaced with a potassium-free solution (eg, albumin). plasmapheresis?
748
Hypokalemia Related to Transcellular Shift of Potassium
How is the concentration gradient The concentration gradient of K+ between the intracellular and extracellular fluid compartments is of K+ between the intracellular and maintained by the Na /K -adenosine triphosphatase (ie, Na /K -ATPase or Na /K+ pump), which uses + + + + + extracellular fluid compartments energy to move K+ against its concentration gradient from the extracellular to the intracellular maintained? compartment (see Figure 35-4).3 What are the chief factors that Under normal conditions, insulin and catecholamines are the primary drivers of transcellular shifts affect transcellular shifts of K+? of K . Acid-base derangements and plasma tonicity also affect transcellular shifts of K+ (see Figure + 35-4).3
What are the causes of hypokalemia related to transcellular shift?
The acid-base state of blood Alkalemia. associated with Bartterâs and Gitelmanâs syndromes. Stress, exercise, and Adrenergic excess. medications. This hormone is often used as Insulin. treatment for hyperkalemia. Often induced therapeutically Hypothermia. in some cases of cardiac arrest. A familial disease resulting in Hypokalemic periodic paralysis.12 transient episodes of weakness and paralysis with a male to female predominance of 3:1.
In addition to being a cause of hypokalemia itself, alkalemia is associated with what other causes of hypokalemia? What are the effects of catecholamines on the Na /K+ + pump? What is the effect of insulin on the Na /K+ pump? + What syndrome can result in insulin-mediated cellular influx of K+ after nutrition is given to a patient following a period of prolonged malnutrition? What dangerous electrolyte disturbance can occur in hypothermic patients who are supplemented with K+ and subsequently rewarmed? How common is hypokalemic periodic paralysis?
Alkalemia is associated with mineralocorticoid excess, RTA, Gitelmanâs syndrome, Bartterâs syndrome, and excess GI loss (eg, vomiting) and can contribute to the hypokalemia generated by those conditions. Hypokalemia can be an important factor in the maintenance of alkalemia by impairing excretion and augmenting H+ excretion in the kidney.13 β-Adrenergic receptors activate the Na /K+ pump, whereas Îą-adrenergic receptors impair cellular entry of + K+ (see Figure 35-4).3 Insulin increases the activity of the Na /K+ pump, thereby accelerating the movement of K+ into the + intracellular compartment (see Figure 35-4).3 Refeeding syndrome is associated with insulin-mediated cellular influx of K . +
Hyperkalemia can occur in patients with hypothermia during rewarming, as there is rapid efflux of K+ from the intracellular to the extracellular compartment.14
The prevalence of hypokalemic periodic paralysis in the industrialized world is approximately 1 in 100,000 persons. It is transmitted in an autosomal dominant pattern and usually presents within the second decade of life. Factors that may trigger episodes of paralysis include emotion, stress, cold exposure, and alcohol ingestion. Attacks occur more often at night and are characterized by flaccid paralysis of all four limbs. The associated hypokalemia may be profound, with serum K+ levels as low as 1 mEq/L.12
749
750 Case Summary A 43-year-old woman with a history of kidney stones presents with subacute weakness and myalgias, chronic dry eyes, dry mouth, and
vaginalldryness, and istfound to have severe hypokalemia and other What is the most likely cause of hypokalemia in this patient? Renal tubular acidosis type 1 (distal). metabo ic derangemen s.
751 Bonus Questions
What is renal tubular RTA describes atgroup of conditions in which there is impaired capacitytfor urinary acidification despite relatively preserved glomerular filtration, resulting acidosis? in net acid reten ion, non-anion gap metabolic acidosis, and various elec rolyte disturbances. The 3 major forms include RTA type 1 (distal), RTA type 2 (proximal), and RTA type 4 (ie, hypoaldosteronism). The 2 forms of distal RTA (types 1 and 4) occur as a result of impaired H excretion, whereas RTA type + 2 occurs as a result of impaired bicarbonate reabsorption.8 What is the significance o In the setting of acidemia, renal acid fication of urine (measured by urine pH) can be used as a marker of renal compensatory capacity. In general, the urine pH in this case? f appropriate compensation results iniurine pH <5.3. If the urine pH isi>5.5 (as in this case) it suggests impaired renal acidification of urine, consistent with What is the significance of UAG =( +) âClâ . Ammonium () is the major unmeasured cation in urine. In the setting of acidemia and normal renal function, there is a compensatory gap (UAG) in this case? intact renal excretiont(eg, diarrhea). If the UAG isrpositive, as in thislcase, then urinetis low, indicatingtimpairment in excretion (eg, distal RTA). The UAG is Which additional The presence of hypokalemia in this case is suggestive of RTA type 1. RTA type 4, on the other hand, is associated with hyperkalemia.8 RTA. Urine pH >5.5 is typical of RTA type 1 but is a less reliable find ng in other types of RTA. 8,15 urine increase in acid elimination by the kidney via exc etion, which is ref ected by a nega ive UAG. A nega ive UAG therefore indicates a source of acidemia with the positive urine anion typically negative in he setting of RTA type 2 because distal acidification is intact. The UAG can be helpful in differentiating proximal and distal RTAs. 8 laboratory finding in this case is suggestive of renal tubular acidosis type 1 as opposed to type 4? What finding is present on The electrocardiogram in this case (see Figure 37-1) demonstrates prominent U waves (particularly in the precordial leads V2 and V3), a characteristic the electrocardiogram in electrocardiographic feature of hypokalemia. this case? What is the mechanism of The chronic metabolic acidosis of RTA type 1 increases the flow rate and delivery of Na+and water to the distal nephron through a variety of mechanisms. This leads to volume depletion, which stimulates renin and aldosterone secretion. Increased Na+delivery and elevated aldosterone levels are potent hypokalemia in patients with renal tubular acidosis type 1? stimulants of K+secretion in the distal nephron, leading to hypokalemia.1 How much K cshould be In patients with normal renal function, 10 mEq of K+would be expected to raise the serum K+concentration by approximately 0.1 mEq/L. + given to corre t hypokalemia? What is the significance of RTA type 1 is associated with hypercalciuria, hypocitraturia, and alkaline urine. Such an environment promotes the formation of calcium phosphate the history of kidney stones. 16 stones in this case? What is the logy of renal RTA type 1 is caused by inherited or acquired defects of the Îą-intercalated cells of the collecting duct (eg, impaired activity of the luminal H -ATPase) that + pathophysio tubular acidosis type 1? retard H excretion. + 8 What is the treatment for Because urinary acid excretion is impaired in patients with RTA type 1, exogenous alkali therapy is necessary to balance daily acid production. Sod um renal tubular acidosis type bicarbonate or sodium citrate is typically used. Citrate salts have the added benefit of correcting hypocitraturia, which can prevent the formation ofikidney 1? stones. Correction of the underlying metabolic acidosis usually corrects the associated hypokalemia. However, some patients may require chronic K + supplementation.17 What is the significance of In this case, the history of dry eyes, dry mouth, vaginal dryness, and minimal tear production on Schirmerâs test are indicative of SjĂśgrenâs syndrome, which and minimal tear the dry mucous membranes can be associated with RTA type 1. 8 production in this case?
752 Key Points
exogenous intake, excretion in urine and stool, and transcellular Maintenance of normal serum K+ concentration is important for Hypokalemia is generally defined as serum K+ <3.6 mEq/L. weakness, myalgias, cramps, constipation, electrocardiographic Potassium homeostasis is regulated by the interplay between shifts. the stability of heart and muscle cells. Clinical manifestations of hypokalemia include malaise, muscle changes (eg, U waves), and dysrhythmia. Hypokalemia can be caused by low intake, excess loss, or transcellular shift of K . Excess K loss can be renal or extrarenal in nature. + + Transcellular shift of K+ occurs rapidly and is primarily driven by insulin and catecholamines. + In patients with normal renal unction, 10 mEq of K would be +
expected to raise the serum Kfconcentration by approximately 0.1 mEq/L.
753
References 1. AronsoniPS, Giebisch G. Effects of pH on potassium: new explanations for old 2. El-Sherif N, Turitto G. Electrolyte disorders and arrhythmogenesis. Cardiol J. observat ons. J Am Soc Nephrol. 2011;22(11):1981-1989. 2011;18(3):233-245.
-
Palmer BF. Regulation of potassium homeostasis. Clin J Am Soc Nephrol. 2015;10(6):1050-1060.
-
Veltri KT, Mason C. Medication-induced hypokalemia. P T. 2015;40(3):185-190.
-
Agarwal R, Afzalpurkar R, Fordtran JS. Pathophysiology;of potassium absorption and 6. Whang R, Whang DD, Ryan MP. Refractory potassium repletion. A consequence of secretion by the human intestine. Gastroenterology. 1994 107(2):548-571. magnesium deficiency. Arch Intern Med. 1992;152(1):40-45.
-
Mohr JA, Clark RM, Waack TC, Whang R. Nafcillin-associated hypokalemia. JAMA. 1979;242(6):544.
-
Reddy P. Clinical approach to renal tubular acidosis in adult patients. Int J Clin Pract. 2011;65(3):350-360.
-
Ji W, Foo JN, OâRoak BJ, et al. Rare independent mutations in renal salt handling genes contribute to blood pressure variation. Nat Genet. 2008;40(5):592-599. . Knochel JP, Dotin LN, Hamburger RJ. Pathophysiology of intense physical
10conditioning in a hot climate. I. Mechanisms of potassium depletion. J Clin Invest. 1972;51(2):242-255. . Kim HW, Chang JH, Park SY, et al. Factors associated with hypokalemia in
11continuous ambulatory peritoneal dialysis patients. Electrolyte Blood Press. 2007;5(2):102-110.
- Fontaine B, Lapie P, Plassart E,:et al. Periodic paralysis and voltage-gated ion channels. Kidney Int. 1996;49(1) 9-18.
-
Palmer BF. Evaluation and treatment of respiratory alkalosis. Am J Kidney Dis. 2012;60(5):834-838.
-
Zydlewski AW, Hasbargen JA. Hypothermia-induced hypokalemia. Mil Med. 1998;163(10):719-721.
-
Yaxley J,.Pirrone C. Review of the diagnostic evaluation of renal tubular acidosis. Ochsner J 2016;16(4):525-530.
- Pereira PC, Miranda DM, Oliveira EA, Silva AC..Molecular pathophysiology of renal tubular acidosis. Curr Genomics. 2009;10(1):51-59
- Batlle D, Haque SK.lGenetic causes and mechanisms of distal renal tubular acidosis. Nephrol Dial Transp ant. 2012;27(10):3691-3704.
754
CHAPTER 38
755
Hyponatremia
756 Case: A 68-year-old man with hemoptysis A previously healthy 68-year-old man presents to the clinic after
coughing up streaks of blood over the past week. He has experienced insisted he maintain his normal food and fluid intake. He smokes 1.5 mild nausea and malaise over the same period of time. His wife has packs of cigarettes per day, a nearly lifelong habit. Heart rate is 80 beats per minute, blood pressure is 123/81 mm Hg, axillae are moist. Jugular venous pressure is estimated to be 6 cm H2O. and respiratory rate is 22 breaths per minute. Mucous membranes and There is no peripheral edema. + Serum sodium (Na ) is 120 mEq/L; blood urea nitrogen ( 13 mg/dL; serum creatinine, 0.8 mg/dL; serum osmolality, BUN), 264 mOsm/kg; urine Na , 60 mEq/L; and urine osmolality, + 620 mOsm/kg. Contrast-enhanced computed tomography imaging of the chest
with coronal (Figure 38-1A) and axial (Figure 38-1B) views reveals a right hilar mass measuring 9.5 Ă7.5 cm with encasement of the right main bronchus and bronchus intermedius; there are associated
enlarged right paratracheal, subcarinal, and supraclavicular lymph nodes.
FIGURE 38-1
What is the most likely cause of hyponatremia in this patient?
What is hyponatremia? What is normal serum Na+ concentration? How is the severity of hyponatremia defined? How common is hyponatremia? How is Na+ normally distributed within the body? How is water homeostasis
Hyponatremia is defined as low serum Na+ concentration that occurs when there is an excess of water relative to Na+ in the extracellular fluid compartment.1 Normal serum Na+ is 135 to 142 mEq/L.2
Severity of hyponatremia is variably defined, but the following thresholds provide a rule of thumb: serum Na+ 130 to 135 mEq/L is mild, serum Na+ 125 to 129 mEq/L is moderate, and serum Na+ <125 mEq/L is severe.1
Hyponatremia is one of the most common electrolyte abnormalities in hospitalized patients, with at least mild hyponatremia occurring in up to one-fifth of patients.1 Most Na+ resides in extracellular fluid, and its distribution between extracellular and intracellular fluid is regulated by the Na /K -adenosine triphosphatase (ie, Na /K -ATPase or Na /K+ pump).3 + + + + +
The central nervous system and the kidneys work in concert to maintain water homeostasis, which ultimately controls serum Na+ concentration. Osmoreceptors located in the hypothalamus detect extracellular tonicity and respond by
757
regulated?
What are the effects of serum Na+ concentration on cell volume? What are the effects of serum Na+ concentration on brain tissue? What adaptive mechanisms occur in the setting of chronic hyponatremia?
adjusting thirst and the secretion of vasopressin (ie, arginine vasopressin [AVP], or antidiuretic hormone [ADH]). of Vasopressin acts in the kidney to increase the reabsorption of free water; it can concentrate the urine to a maximum 1200 mOsm/kg. In the absence of vasopressin, urine osmolality can drop to a minimum of 50 mOsm/kg. In the setting of hyponatremia, thirst and vasopressin secretion should be inhibited. A significant decrease in blood volume also stimulates vasopressin secretion (see Figure 36-1).2 Sodium is an effective solute and therefore contributes to overall serum â¨tonicity. Water moves freely between fluid compartments in response to differences in tonicity. Accordingly, the presence of hypernatremia, which is always associated with hypertonicity, will result in a shift of water out of cells, causing cellular contraction. Conversely, hypotonic hyponatremia will result in a shift of water into cells, causing cellular swelling (see Figure 36-2).3 In most tissues of the body, the Na+ concentrations of serum and interstitial fluid are virtually identical due to free movement of Na+ through the capillary membrane. In contrast, capillaries in the brain are impermeable to Na . The result is that an abnormal serum Na+ concentration will cause water to move into or out of brain tissue, with subsequent + swelling or contraction, respectively. Only minimal changes in the volume of brain tissue are compatible with life.2 In the setting of chronic hypotonic hyponatremia, defined by at least 24 to 48 hoursâ duration, osmotically active intracellular molecules (osmolytes) (eg, glutamate, taurine, myo-inositol) are leaked out of the cell. This helps decrease the difference in tonicity between extracellular and intracellular fluid, which in turn, decreases movement of water into cells. This adaptation is particularly important in brain tissue (Figure 38-2).1,2
FIGURE 38-2 Schematic diagram of brain volume adaptation to hyponatremia. Under normal conditions brain tonicity and
After correction of hypotonicity, how long does it take for the recovery of lost intracellular osmolytes to occur? What are the clinical differences between acute and chronic hyponatremia? What are the clinical manifestations of acute hyponatremia? What is the first step in establishing the cause of hyponatremia?
What are the laboratory definitions of hypertonicity,
extracellular fluid tonicity are in equilibrium (top). Following the induction of extracellular fluid hypotonicity, water moves into the brain, producing brain edema (dotted line, middle, #1). However, in response to the induced swelling, the brain rapidly loses intracellular solutes (middle, #2). As water losses accompany the losses of brain solute, the expanded brain volume then decreases back toward normal (middle, #3). If hypotonicity is sustained, brain volume eventually normalizes completely, and the brain becomes fully adapted to hyponatremia (bottom). (From Schrier RW. Diseases of the Kidney and Urinary Tract. 8th ed. Philadelphia, PA: Lippincott Williams & Wilkins; 2007.)
After chronic hypotonicity is corrected, the recovery of lost intracellular electrolytes can take a week or longer.2
Given the adaptation that occurs in response to chronic hyponatremia, patients are often asymptomatic. In contrast, patients with acute hyponatremia (developed over a period of hours) are almost always symptomatic, and changes in brain volume can be life-threatening. The physiologic adaptation that occurs in chronic hyponatremia limits the rate with which hyponatremia can be corrected (see Figure 36-3). In cases where duration is unknown, chronic hyponatremia should be assumed.2 Clinical manifestations of acute hyponatremia may include nausea, malaise, lethargy, headache, delirium, obtundation, seizure, and coma.2
Determining serum tonicity is the first step in establishing the cause of hyponatremia.
Hypertonicity is present when the effective serum osmolality is >295 mOsm/kg, hypotonicity is present when the effective serum osmolality is <275 mOsm/kg, and isotonicity is present when the effective serum osmolality is 275 to 295 mOsm/kg.4
758
isotonicity, and hypotonicity? What is the Serum osmolality takes into account all solutes, including those that are effective (ie, solutes that do not freely move difference between the intracellular and extracellular fluid compartments) and those that are ineffective (ie, solutes that move between serum freely). In contrast, serum tonicity (also called effective osmolality) takes into account only effective solutes. osmolality and serum tonicity? What is the Ineffective solutes contribute to serum osmolality, but move freely between intracellular and extracellular fluid clinical compartments to maintain an even concentration gradient and, therefore, do not generate an osmotic gradient. Without difference an osmotic gradient, movement of water between extracellular and intracellular fluid compartments does not occur. between Effective solutes, in contrast, do not move freely, giving rise to osmotic gradients that trigger movement of water. effective and ineffective solutes? What are the The main contributors to serum osmolality include serum Na , glucose, and BUN. + main contributors to In the above formula, [Na+] is measured in mEq/L or mmol/L, and [glucose] and [BUN] are measured in mg/dL.5 serum osmolality? Are Na , Sodium and glucose are effective solutes, whereas urea is an ineffective solute. + glucose, and urea effective or ineffective solutes? How could the To better reflect serum tonicity, the contribution of urea should be subtracted from the formula for serum osmolality formula for because it is an ineffective solute.1 serum osmolality be modified to better reflect serum tonicity?
759
Hypertonic and Isotonic Hyponatremia
What are the causes of hypertonic or isotonic hyponatremia?
A common cause of hypertonic hyponatremia that is not associated with a serum osmolal gap. This agent may be used to acutely reduce intracranial pressure. Laboratory artifact.
What is the expected decrease in serum Na+ concentration caused by hyperglycemia? What are the effects on cell volume of hyperosmolar hyponatremia related to either hyperglycemia or hypertonic mannitol? Are there any circumstances in which hyperosmolar hyponatremia may be associated with cellular swelling? What clinical conditions are associated with pseudohyponatremia?
Hyperglycemia.
Hypertonic mannitol.
Pseudohyponatremia.
For each 100 mg/dL increase in serum glucose concentration above 100 mg/dL, the serum Na+ decreases by 1.6 to 2.4 mEq/L. Using a correctional factor of 2.0 mEq/L, the following formula can be used to determine the value that should be added to the measured serum Na+ concentration1: ([measured glucose â100]/100) Ă2.0 The hyperosmolality associated with hyperglycemia and hypertonic mannitol results in cellular contraction. Because glucose and mannitol are effective solutes, the resultant high serum tonicity acts to draw water out of cells and into the extracellular fluid compartment. This dilutes the serum Na+ concentration, resulting in hyponatremia. Serum hyperosmolality related to ineffective solutes (eg, urea, alcohol) does not trigger the movement of water into the extracellular fluid compartment. If there is coexistent hyponatremia, it must be driven by an unrelated process, which could include those associated with serum hypotonicity. If overall serum tonicity is lower than intracellular tonicity, water will move into the intracellular fluid compartment, causing cellular swelling. Pseudohyponatremia is associated with hyperlipidemia and paraproteinemia.1
760
Hypotonic Hyponatremia
What is the first step in establishing the cause of hypotonic hyponatremia?
What physical findings are associated with hypovolemia? What physical findings are associated with hypervolemia? What laboratory test can be helpful in distinguishing hypovolemic patients from euvolemic patients?
Determining extracellular fluid volume status is the first step in establishing the cause of hypotonic hyponatremia.
Hypovolemia may be associated with Decreased skin turgor, dry mucous membranes and axillae, Increased capillary refill time, sunken eyes, and low jugular venous pressure. Hypervolemia may be associated with elevated jugular venous pressure, ascites, rales on lung auscultation (due to pulmonary edema), dullness to percussion of the lung bases (due to pleural effusions), and peripheral edema. In patients who are not hypervolemic, spot urine Na+ concentration can be helpful in distinguishing between hypovolemia (UNa <30 mEq/L) and euvolemia (UNa >30 mEq/L). Cases of hypovolemia due to renal Na+ wasting (eg, primary adrenal insufficiency, diuretic use) can be associated with relatively higher UNa, possibly exceeding 30 mEq/L.6
761
Hypovolemic Hyponatremia Hypovolemic hyponatremia can be thought of as extracellular loss of hypertonic fluid (salt >water).
What is the status of vasopressin levels in patients with hypovolemic hyponatremia? What are the 2 subcategories of hypovolemic hyponatremia?
Which laboratory test can be helpful in distinguishing renal from extrarenal causes of hypovolemic hyponatremia?
Decreased effective arterial blood volume (reduced by at least 10%-20%) with inadequate circulation results in baroreceptor-mediated release of vasopressin, which acts to maintain intravascular volume at the expense of serum tonicity (see Figure 36-1).2 Hypovolemic hyponatremia can be caused by renal or extrarenal processes.
Spot urine Na+ concentration can be suggestive of whether the source of volume depletion is renal (UNa >20 mEq/L) or extrarenal (UNa <20 mEq/L, especially <10 mEq/L).7,8
762
Renal Causes of Hypovolemic Hyponatremia
What are the renal causes of hypovolemic hyponatremia?
Iatrogenic loss of hypertonic fluid (relative to serum). Hyponatremia, hyperkalemia, and hypotension. A 34-year-old woman with a subarachnoid hemorrhage related to a motor vehicle accident develops hypovolemic hyponatremia.
What is the mechanism of hyponatremia related to thiazide diuretics? What is the treatment for hyponatremia related to adrenal insufficiency? What is cerebral salt wasting?
Diuretic medication, particularly thiazide diuretics.
Primary adrenal insufficiency.
Cerebral salt wasting.
Thiazide diuretics decrease the reabsorption of Na+ and chloride in the distal convoluted tubule (within the renal cortex), which ultimately impairs the ability of the kidneys to make dilute urine (see Figure 36-4). If the osmolality of the excreted urine exceeds that of serum, then hyponatremia will eventually occur. In addition, intravascular hypovolemia related to diuresis stimulates the secretion of vasopressin, which may contribute to the development of hyponatremia. Patients with diuretic-induced hyponatremia may be euvolemic or hypervolemic.9 Hyponatremia related to adrenal insufficiency should be treated with cortisol to decrease vasopressin release, and intravenous fluids to improve the effective arterial blood volume.10
Cerebral salt wasting occurs in the setting of central nervous system disease, particularly subarachnoid hemorrhage. It is characterized by hypovolemic hyponatremia, which develops as a result of renal Na+ wasting. The pathogenesis is poorly understood. The timing of cerebral salt wasting can be variable, with some cases occurring within days of head injury and others occurring up to 2 months later. It may be confused with the syndrome of inappropriate antidiuretic hormone secretion (SIADH), but distinguishing between the two is important as treatment differs. For example, unlike SIADH, cerebral salt wasting should not be treated with water restriction. Volume repletion with isotonic saline is the treatment of choice.11
763
Extrarenal Causes of Hypovolemic Hyponatremia
What are the extrarenal causes of hypovolemic hyponatremia?
You cannot lose what you do not have. True isotonic fluid loss, resulting in Decreased effective arterial blood volume.
Which groups of adults are at Increased risk for low oral intake? What are some causes of isotonic or hypertonic fluid loss from the Gastrointestinal (GI) tract?
Poor oral intake. Gastrointestinal losses.
Adult patients at risk for low oral intake include the elderly, alcoholics, the economically disadvantaged, and patients with eating disorders. Volume repletion with isotonic saline is the treatment of choice. Patients may be at risk for refeeding syndrome when oral intake resumes or increases. Loss of isotonic or hypertonic fluid (relative to serum) from the GI tract can occur as a result of diarrhea, vomiting, or tube drainage. Volume repletion with isotonic saline is the treatment of choice.
764
Euvolemic Hyponatremia Euvolemic hyponatremia can be thought of as extracellular gain of pure water.
What is the status of vasopressin levels in patients with euvolemic hyponatremia?
Which laboratory test can be helpful in establishing whether euvolemic hyponatremia is related to a vasopressin-dependent or vasopressin-independent process?
In euvolemic patients with hyponatremia, vasopressin is either present (vasopressin- dependent) or suppressed (vasopressin-independent), depending on the underlying cause.
Urine osmolality can be suggestive of whether euvolemic hyponatremia is vasopressin-mediated or not. In the setting of hyponatremia, urine osmolality >100 mOsm/kg is consistent with a vasopressin-dependent process. In general, urine osmolality greater than serum osmolality nearly always indicates the presence of vasopressin.1
765
Vasopressin-Dependent Causes of Euvolemic Hyponatremia
What are the vasopressin-dependent causes of euvolemic hyponatremia?
A 22-year-old woman Syndrome of inappropriate antidiuretic hormone secretion. with epilepsy is started on a new antiepileptic medication and is subsequently found to have a serum Na+ concentration of 122 mEq/L. She is euvolemic with a urine osmolality of 560 mOsm/kg. Endocrinopathies. Hypothyroidism and secondary adrenal insufficiency. In patients with this Reset osmostat. condition, thirst and vasopressin secretion may be stimulated at a serum Na+ concentration >132 mEq/L, and inhibited at values <132 mEq/L.
What are the causes of syndrome of inappropriate antidiuretic hormone secretion? Under what conditions is hypothyroidism associated with hyponatremia? Why does secondary adrenal insufficiency cause euvolemic hyponatremia rather than hypovolemic hyponatremia (as seen in primary adrenal insufficiency)? What are the characteristics of reset osmostat?
Causes of SIADH include medication (eg, antipsychotics), lung disease (eg, pneumonia), malignancy (eg, small cell lung cancer), central nervous system disease (eg, brain tumor), pain, nausea, and stress.1
Although it is rare, hyponatremia can develop in patients with severe hypothyroidism (particularly myxedema coma). As a result of its effects on cardiac output and systemic vascular resistance, severe hypothyroidism causes a decrease in renal perfusion and an increase in baroreceptor-mediated vasopressin secretion, leading to impaired renal free water excretion (see Figure 36-1). The association between hypothyroidism of lesser severity and hyponatremia is not as clear.12 The hypocortisolism characteristic of both primary and secondary adrenal insufficiency leads to a loss of hypothalamic inhibition, with an associated increase in corticotropin-releasing hormone from the hypothalamus, which is a vasopressin secretagogue. The Increased vasopressin levels impair renal free water excretion, leading to hyponatremia. In primary adrenal insufficiency, direct involvement of the adrenal cortex may lead to an additional deficiency of mineralocorticoids. Hypoaldosteronism results in renal Na+ and water wasting, leading to hypovolemia and baroreceptor-mediated stimulation of vasopressin (ie, hypovolemic hyponatremia). In secondary adrenal insufficiency, mineralocorticoid activity remains intact because it is not dependent on adrenocorticotropic hormone and is instead under the control of the renin-angiotensin system. While isolated hypocortisolism may lead to hyponatremia, sufficient levels of aldosterone in these patients prevent renal Na+ and water wasting, thereby maintaining euvolemia (ie, euvolemic hyponatremia).4,13 Reset osmostat describes a serum osmotic threshold (the level at which thirst and vasopressin secretion are stimulated and inhibited) that is lower than normal. The resultant hyponatremia is typically chronic, mild to moderate, and asymptomatic. Reset osmostat is technically a subtype of SIADH, but treatment is usually not necessary since thirst and vasopressin secretion are inhibited once the lower osmotic threshold is reached, which halts any further reduction in serum Na .4,8 +
766
767
Vasopressin-Independent Causes of Euvolemic Hyponatremia
What are the vasopressin-independent causes of euvolemic hyponatremia?
Commonly associated with psychiatric disorders. Low solute intake.
What is primary polydipsia?
What is the mechanism of hyponatremia related to beer potomania?
Primary polydipsia.
Beer potomania (ie, âtea and toastâ potomania).
Hyponatremia caused by primary polydipsia occurs most often in patients with psychiatric disorders, particularly schizophrenia. It develops when water ingestion overwhelms renal excretory capacity. The capacity of the kidney to excrete free water is dependent on solute excretion and urinary diluting capability. A typical Western diet results in solute excretion of around 800 mOsm/d. The maximum urinary dilution capability of healthy kidneys is 50 mOsm/L. Therefore, the maximum volume of free water that can be excreted in a day is around 16 liters (800 mOsm/day / 50 mOsm/L =16 L per day). If water ingestion exceeds this threshold (as in primary polydipsia), then the resultant free water excess will cause hyponatremia. Primary polydipsia is treated with free water restriction. Associated mental health disorders should also be addressed.4,14 Low solute intake can occur in a variety of settings, most often affecting patients who drink large volumes of beer with little food intake and those on low protein diets. Low solute excretion limits the volume of urinary free water that can be removed from the body. For example, in a patient with 250 mOsm of obligatory solute excretion per day, assuming maximal urinary dilution of 50 mOsm/L, the maximum daily volume of urinary free water excretion is 5 liters (250 mOsm/d / 50 mOsm/L =5 L per day). If the daily ingestion of free water exceeds 5 liters (14 cans of beer), then hyponatremia will develop. These patients must be monitored carefully as an Increased solute load will result in brisk diuresis and dangerously rapid correction of hyponatremia.14
768
Hypervolemic Hyponatremia Hypervolemic hyponatremia can be thought of as extracellular gain of hypotonic fluid (water >salt).
What is the status of vasopressin levels in patients with hypervolemic hyponatremia? What are the 2 subcategories of hypervolemic hyponatremia?
Despite the presence of total body hypervolemia, the conditions that lead to hypervolemic hyponatremia are associated with a decrease in effective arterial blood volume, resulting in baroreceptor-mediated release of vasopressin, which acts to maintain intravascular volume at the expense of serum tonicity. Hypervolemic hyponatremia can be caused by renal or extrarenal processes.
769
Renal Causes of Hypervolemic Hyponatremia
What are the renal causes of hypervolemic hyponatremia?
Asterixis and a pericardial friction rub. Associated with anasarca and foamy urine.
What is the mechanism of hyponatremia related to renal failure? What are the characteristic urinary findings of nephrotic syndrome?
Renal failure.
Nephrotic syndrome.
In advanced renal failure, the capacity of the kidneys to dilute urine becomes impaired, such that minimum urine osmolality may be as high as 200 to 250 mOsm/kg. This chronic impairment in free water excretion eventually leads to free water retention and hyponatremia.15
Nephrotic syndrome is characterized by proteinuria of at least 3.5 g/d. A minority of patients may also experience microscopic hematuria, but the urine sediment is typically bland. Hyponatremia is not common early in the course of nephrotic syndrome. However, when serum albumin concentration falls below 2 g/dL, intravascular volume depletion may stimulate vasopressin secretion, followed by the development of hyponatremia.4,16
770
Extrarenal Causes of Hypervolemic Hyponatremia
What are the extrarenal causes of hypervolemic hyponatremia?
Poor forward blood flow to the kidneys. A middle-aged man with a history of alcohol abuse is found to have hypervolemic hyponatremia associated with spider angiomas, flank fullness, and asterixis.
Is diuresis effective in the treatment for hyponatremia associated with heart failure?
Is diuresis effective in the treatment for hyponatremia associated with cirrhosis?
Heart failure. Cirrhosis.
Diuretics often improve hyponatremia associated with heart failure by optimizing preload, which improves cardiac output and effective arterial â¨blood volume, thereby inhibiting baroreceptor-mediated vasopressin release. However, diuretic therapy, particularly thiazide agents, may worsen hyponatremia in some patients.17 Hyponatremia is common in patients with cirrhosis, but only rarely occurs in the absence of ascites. It develops when vasodilation of the splanchnic circulation results in baroreceptor-mediated vasopressin secretion. Diuretics, including loop and thiazide agents, often exacerbate the hyponatremia associated with cirrhosis because these medications contribute to intravascular hypovolemia, stimulating the secretion of vasopressin.18
771 Case Summary A 68-year-old man with an extensive smoking history presents with
nausea, malaise, and hemoptysis and is found to have hyponatremia, elevated urine osmolality, and a lung mass on chest imaging.
What is the most likely cause of hyponatremia in this patient? Syndrome of inappropriate antidiuretic hormone secretion.
772
Bonus Questions
Which general categories This case canibe described as hypotonic, euvolemic, vasopressin-dependent hyponatremia. Categorizing hyponatremia in this way significantly narrows the to this case? of hyponatremia apply differential d agnosis. What features in this The normal jugular venous pressure and absence of peripheral edema in this case effectively rule out hypervolemia. Euvolemia and hypovolemia can be
case are suggestive of difficult to differentiate clinically. However, there is no history of poor oral intake or excess fluid loss tolsuggest hypovolemia. Moreover, the presence of moist What information in this In the setting of hyponatremia, urine osmolality >100 mOsm/kg is indicative of a vasopressin-mediated process.1 status? euvolemic volume mucous membranes and axillae, low BUN, and urine Na concentration >30 mEq/L suggest that extrace lular fluid volume is not low. + 1 case indicates that the hyponatremia is vasopressin-dependent? What is the most likely Lung disease is a common cause of SIADH; in this case, given the smoking history, lung mass (see Figure 38-1), and evidence of SIADH, the most likely syndrome of underlying cause of diagnosis is lung cancer. Small cell lung cancer in particular is most often associated with SIADH. 1 inappropriate antidiuretic hormone secretion in this case? Is the hyponatremia in In this case, chronic hyponatremia is suggested by the lackiof.symptoms. In cases where the duration of hyponatremia is unknown, chronicity should be What is the definitive Elimination of the underlying cause is the definitive therapy for SIADH; most cases related to malignancy resolve with effective antineoplastic therapy.1 syndrome of acute or chronic? this case most likely presumed, which has implications on management strateg es management of inappropriate antidiuretic hormone secretion? In addition to addressing Given that the hyponatremia in this case is most likely chronic, careimust be taken not to treat too aggressively. In cases of chronic hyponatremia, a rate of what other management strategies should be the underlying cause, correction of no more than 0.5 to 1 mEq/L per hour is reasonable (w th a maximum of 8 mEq/L per 24-hour period); close monitoring of the serum Na + considered in this case? concentration isfprudent. Fluid restriction is key to treating chronic hyponatremia related to SIADH. Other options include the use of salt tablets, hypertonic saline infusion, urosemide, urea, and vasopressin antagonists. 1,2 Why is it dangerous to Within 24 to 48 hours of the onset of hypotonic hyponatremia, brain tissue begins to adapt to increases in water content and associated cerebral edema. One correct chronic such adaptation is the leakage of osmolytes from brain cells, which acts to reduce the tonicity gradient between the extracellular and intracellular Figure 38-2 hyponatremia too environments, mitigating the influx of water (see ). Once this compensatory mechanism has occurred, rapid correction of the serum Na+ concentration will resultiin an abrupttdifference in tonicity between extracellular and intracellular fluid, favoring the movement of water out of the cells, quickly? which can lead to osmot c demyelina ion, a devastating complication. 2 What are the clinical Osmotic demyelination is a biphasic syndrome, with initial manifestations typically occurring days after overcorrection of hyponatremia (there may be a lag of manifestations of up to a week in some cases). Neurologic symptoms may improve initially, but this is followed by the gradual onset of new and variable neurologic osmotic demyelination? manifestations, including seizures, behavioral abnormalities, andimovement disorders. In.severe cases, when there is involvement of the pons, patients s develop a âlocked-inâ syndrome with quadriparesis and the inab lity to speak or swallow Patients may recover from osmotic demyelination, sometime completely, but many others develop permanent disability or die. 2,19 What is the management For symptomatic and severe acute hyponatremia (<24-48 hours), rapid treatment is imperative. This can be achieved by infusing 3% saline intravenouslyiwith prudent. Once the neurologic symptoms have abated (usually after the initial increase in serum Na+of 4-6 mEq/L), the rate can generally be slowed.1,2 hyponatremia? + of symptomatic acute a goal rate of correction of 1 to 2 mEq/L per hour (with a maximum of 8-10 mEq/L per 24-hour period); close monitoring of the serum Na concentration s
773 Key Points
+ Water homeostasis, which controls serum Na concentration, is regulated by thirst and the hormonal interplay between the central nervous system and the kidneys.
Maintenance of normal serum Na+ concentration is important for preserving cell volume. Hyponatremia is generally defined as serum Na <135 mEq/L.
+ Clinical manifestations of hyponatremia depend on its duration and severity. Acute hyponatremia is defined by duration <24 to 48 hours.
Acute hyponatremia is often symptomatic with manifestations that obtundation, seizure, and coma. include nausea, malaise, lethargy, headache, delirium, Chronic hyponatremia is often asymptomatic. Hyponatremia can be associated with serum hypertonicity, Hypotonic hyponatremia can be associated with extracellular Hypovolemic hyponatremia results from extracellular loss of Hypovolemic hyponatremia can be caused by renal or extrarenal Euvolemic hyponatremia results from extracellular gain of pure Euvolemic hyponatremia can be caused by vasopressin-dependent Hypervolemic hyponatremia results from extracellular gain of Hypervolemic hyponatremia can be caused by renal or extrarenal isotonicity, or hypotonicity. hypovolemia, euvolemia, or hypervolemia. hypertonic fluid (salt >water). processes. water. or vasopressin-independent processes. hypotonic fluid (water >salt). processes. Acute symptomatic hyponatremia should be corrected rap dly with infusion of hypertonic saline and serial monitoring ofithe serum Na concentration. Management of chronic hyponatremia depends on the underlying cause but can include fluid restriction salt tablets, hypertonic +
saline infusion, furosemide, urea, and,vasopressin antagonists. The rate of serum Na correction should be carefully considered to + prevent the development of osmotic demyelination.
774
775
References 1. Ellison DH, Berl T. Clinical practice. The syndrome of inappropriate antidiuresis. N Engl J Med. 2007;356(20):2064-2072.
-
Sterns RH. Disorders of plasma sodiumâcauses, consequences, and correction. N Engl J Med. 2015;372(1):55-65. Spital A, Sterns RD The paradox of sodiumâs volume of distribution. Why an
-
extracellular solute.appears to distribute over total body water. Arch Intern Med. 1989;149(6):1255-1257.
-
Verbalis JG, Goldsmith SR, Greenberg A, et al..Diagnosis, evaluation, and treatment of. 5. Berend K, de.Vries AP, Gans RO.;Physiological approach to assessment of acid-base hyponatremia: expert panel recommendations Am J Med. 2013;126(10 suppl 1):S1-S42 disturbances N Engl J Med. 2014 371(15):1434-1445.
-
Chung HM, Kluge R, Schrier RW, Anderson RJ. Clinical assessment of extracellular fluid volume in hyponatremia. Am J Med. 1987;83(5):905-908.
-
Schrier RW. Body water homeostasis: clinical disorders of urinary dilution and concentration. J Am Soc Nephrol. 2006;17(7):1820-1832.
-
Longo DL, Fauci AS, Kasper DL, Hauser SL, Jameson JL, Loscalzo J, eds. Harrisonâs Principles of Internal Medicine. 18th ed. New York, NY: McGraw-Hill; 2012.
-
Ashraf N, Locksley R, Arieff AI. Thiazide-induced hyponatremia associated with death or neurologic damage in outpatients. Am J Med. 1981;70(6):1163-1168. . Ahmed AB, George BC, Gonzalez-Auvert C, Dingman JF. Increased plasma arginine
10vasopressin in clinical:adrenocortical insufficiency and its inhibition by glucosteroids. J 11. Leonard J, Garrett RE, Salottolo K, et al. Cerebral salt wasting after traumatic brain Clin Invest. 1967;46(1) 111-123. injury: a review of the literature. Scand J Trauma Resusc Emerg Med. 2015;23:98.
- Pantalone KM, Hatipoglu BA. Hyponatremia and the thyroid: causality or association? J Clin Med. 2014;4(1):32-36. . van der Hoek J, Hoorn EJ, de Jong GM, Janssens EN, de Herder WW. Severe
13hyponatremia with high urine sodium and osmolality. Clin Chem. 2009;55(11):1905- 1908. . Sanghvi SR, Kellerman PS, Nanovic L. Beer potomania: an unusual cause of
14hyponatremia at high risk of complications from rapid correction. Am J Kidney Dis. 2007;50(4):673-680.
-
Tannen RL, Regal EM, Dunn MJ, Schrier RW. Vasopressin-resistant hyposthenuria in advanced chronic renal disease. N Engl J Med. 1969;280(21):1135-1141.
-
Hull RP, Goldsmith DJ. Nephrotic syndrome in adults. BMJ. 2008;336(7654):1185-1189. 17acute decompensated heart failure: depletion versus dilution. J Am Coll Cardiol.a in . Verbrugge FH, Steels P, Grieten L, Nijst P, Tang WH, Mullens W. Hyponatremi 2015;65(5):480-492.
-
Sherlock S, Senewiratne B, Scott A, Walker JG. Complications of diuretic therapy in hepatic cirrhosis. Lancet. 1966;1(7446):1049-1052.
-
King JD, Rosner MH. Osmotic demyelination syndrome. Am J Med Sci. 2010;339(6):561-567.
776
777
CHAPTER 39
778
Secondary Hypertension
779 Case: A 24-year-old man with discordant peripheral pulses A 24-year-old man presents to the clinic for evaluation of regularly see a physician. His blood pressure was measured at the hypertension. He has no known medical conditions and does not local grocery store, and he was told it was elevated. He does not take any medications, including over-the-counter agents or supplements. f Blood pressure is 182/98 mm Hg in the right upper extremity, Review of systems is notable for headache and exertional shortness o breath over the past few months. 178/94 mm Hg in the left upper extremity, 103/72 mm Hg in the right lower extremity, and 98/64 mm Hg in the left lower extremity..Jugular 2 venous pressure is 8 cm H O. The carotid pulses are bounding The radial pu ses are strong, but the dorsalis pedis pu ses are weakly palpable.lThere is an extra heart sound heard justlbefore S1 with the cl Electrocardiogram is notable for left ventricular hypertrophy. A bell of the stethoscope over the apex. No murmurs are present. ose-up of the chest radiograph is shown in Figure 39-1.
FIGURE 39-1 (From Daffner RH, Hartman MS. Clinical Radiology: The Essentials. 4th ed. Philadelphia, PA: Lippincott Williams & Wilkins; 2014.)
What is the most likely cause of hypertension in this patient?
What is essential hypertension?
What is secondary hypertension?
How common is secondary hypertension? What clinical characteristics are suggestive of secondary hypertension? What is the clinical
Essential hypertension is characterized by a chronic abnormal elevation in systolic or diastolic arterial blood pressure (BP) without a clear underlying etiology. There are likely multiple underlying factors involved in the development of essential hypertension, including genetic and environmental elements (eg, diet).1 Secondary hypertension is defined as abnormal elevation in systolic arterial BP resulting from an identifiable, and often correctable, underlying cause. Primary and secondary hypertension can coexist in the same patient, so some degree of hypertension may persist despite appropriate treatment of secondary causes.2 In the industrialized world, secondary hypertension affects approximately 5% to 10% of the general hypertensive population.2 Any of the following clinical characteristics are suggestive of secondary hypertension: onset of hypertension at a young age (ie, <30-40 years), few risk factors for essential hypertension (eg, family history, obesity), resistant hypertension (BP >140/90 mm Hg) despite several antihypertensive drugs, episodes of severe hypertension (>180/100 mm Hg), abrupt increase in systolic BP in a previously stable patient, labile hypertension, and evidence of target organ damage (eg, left ventricular hypertrophy).1,2 Hypertension is a key risk factor for stroke, myocardial infarction, heart failure, and kidney failure.1
780
significance of How is blood pressure hypertension? regulated? The causes of secondary hypertension can be separated into which general categories?
Moment-to-moment regulation of blood pressure is controlled by the neurally mediated baroreceptors found in the carotid sinus and aortic arch. Long-term regulation of blood pressure is controlled by the hormone-based renin- angiotensin-aldosterone system, and atrial and brain natriuretic peptides (see Figure 22-2).3 The causes of secondary hypertension can be separated into the following categories: vascular, endocrinologic, toxic, and other.
781
Vascular Causes of Secondary Hypertension
What are the vascular causes of secondary hypertension?
A 35-year-old woman with new-onset hypertension and an abdominal bruit on examination. Rib notching on chest radiography. A 43-year-old man with testicular pain, hypertension, and chronic hepatitis B infection.
What are the most common causes of renal artery stenosis in younger and older populations?
What blood pressure finding can be a clue to the diagnosis of coarctation of the aorta? Which types of large and medium vessel vasculitis are associated with secondary hypertension?
Renal artery stenosis.
Coarctation of aorta.
Polyarteritis nodosa.
Renal artery stenosis may be responsible for up to 20% of cases of resistant hypertension. Fibromuscular dysplasia is the most common cause in children and young adults, whereas atherosclerosis is the most common cause in older adults. Clues to the diagnosis include an abdominal bruit (particularly when diastolic) and acute renal function deterioration after starting an angiotensin-converting enzyme inhibitor or angiotensin receptor blocker. Conventional renal angiography (Figure 39-2) is the diagnostic gold standard, but other less invasive imaging studies are available for initial testing, including duplex ultrasonography.2
FIGURE 39-2 Typical arteriographic âstring of beadsâ appearance of fibromuscular dysplasia. (From Schrier RW. Diseases of the Kidney and Urinary Tract. 8th ed. Philadelphia, PA: Lippincott Williams & Wilkins; 2007.)
Coarctation of the aorta can be associated with discordant blood pressure readings between upper and lower extremities.
Secondary hypertension can occur in patients with medium vessel vasculitides, including polyarteritis nodosa and Kawasaki disease, and large vessel vasculitides, including giant cell arteritis and Takayasu arteritis.4
782
783
Endocrinologic Causes of Secondary Hypertension
What are the endocrinologic causes of secondary hypertension?
Weight gain, cold intolerance, and constipation. Hypertension, hypokalemia, and metabolic alkalosis. Anxiety, heat intolerance, weight loss, and tremor. Treatment for this condition often includes intravenous fluids, calcitonin, and bisphosphonates. Central obesity, thin skin, ecchymosis, and osteoporosis. Arthralgias, macroglossia, jaw enlargement, and headache. Flushing, palpitations, headache, chest pain, and perspiration.
How common is hypertension in patients with hypothyroidism? What pattern of electrolyte and metabolic abnormalities is classically associated with primary hyperaldosteronism? What is characteristic about the hypertension associated with hyperthyroidism?
Hypothyroidism.
Mineralocorticoid excess.
Hyperthyroidism.
Hypercalcemia.
Cushingâs syndrome.
Acromegaly.
Pheochromocytoma.
Hypertension is present in around one-fifth of patients with hypothyroidism. Serum thyroid stimulating hormone (TSH) and free thyroxine (T4) are the initial tests of choice. Hypertension usually resolves with thyroid hormone replacement therapy.5 Primary hyperaldosteronism (ie, Connâs syndrome) may be present in up to 10% of hypertensive patients. The most common causes are bilateral idiopathic hyperplasia of the adrenal glands and unilateral aldosterone-⨠secreting adrenal adenoma. Primary hyperaldosteronism is classically â¨associated with mild hypernatremia, hypokalemia, and metabolic alkalosis, but this pattern of metabolic abnormalities is not always present. â¨The initial test of choice is plasma aldosterone (ng/dL) to plasma renin activity (ng/mL/hr) ratio (a ratio >30 is suggestive of the diagnosis).5 Hyperthyroidism is often associated with an isolated elevation in systolic blood pressure (generating a wide pulse pressure). Serum TSH and T4 are the initial tests of choice. β-Blockers may be useful in treating hypertension until there is definitive management of the underlying hyperthyroidism.5,6
What is the mechanism of hypertension related to hypercalcemia? How common is hypertension in patients with Cushingâs syndrome? How common is hypertension in patients with acromegaly?
The main mechanism of hypertension in the setting of hypercalcemia is direct calcium-mediated increase in systemic vascular resistance (including renal vascular resistance). The hypertension usually resolves with management of hypercalcemia. Thiazide agents should be avoided in hypercalcemic patients.7 Hypertension is present in around one-fifth of patients with iatrogenic Cushingâs syndrome; there is a dramatic increase in occurrence among patients with endogenous Cushingâs syndrome (hypertension is present in as many as 95% of patients with ectopic ACTH secretion). In patients with a clinical syndrome compatible with Cushingâs syndrome (see Figure 8-3), the urine free cortisol test, which measures the quantity of free cortisol secreted in the urine in a 24-hour period, is the most reliable confirmatory test.8 Hypertension occurs in around one-third of patients with acromegaly. Most cases of acromegaly are caused by pituitary tumors that secrete growth hormone; other sources of growth hormone secretion include small cell lung cancer and pancreatic cancer. In patients with a compatible clinical syndrome (see Figure 41-4), serum insulin-like growth factor (IGF-1) is the initial test of choice. Hypertension typically improves with management of acromegaly.5,9,10
784
How common is hypertension in patients with pheochromocytoma?
Hypertension is the most common sign of pheochromocytoma, occurring in the vast majority of patients (about 95%). Elevated blood pressure may be sustained or paroxysmal (acute-on-chronic elevations are also common). Initial test options include urinary and plasma fractionated metanephrines and catecholamines. ι-Blockers (eg, prazosin) are the treatment of choice for hypertension in patients with pheochromocytoma. β-Blockers may be added, but only after ι-blockade has been instituted in order to avoid hypertensive crisis caused by unopposed ι- receptor mediated vasoconstriction. 11
785
Toxic Causes of Secondary Hypertension
What are the toxic causes of secondary hypertension?
Cyclooxygenase inhibitors. Iatrogenic Cushingâs syndrome. Used to treat the symptoms of menopause. Stimulant medications. A beverage. Illicit sympathomimetics.
What is the average increase in blood pressure related to the use of nonsteroidal anti-inflammatory drugs? What is the mechanism of glucocorticoid- induced hypertension?
What are the risk factors for hypertension related to exogenous estrogen? What are some sympathomimetic agents?
How much alcohol consumption is associated with increases in blood pressure? What is the mechanism of hypertension related to cocaine?
What complication involving the pulmonary vasculature can develop in patients who abuse cocaine or amphetamines?
Nonsteroidal anti-inflammatory drugs (NSAIDs). Corticosteroids. Estrogens.
Sympathomimetics. Alcohol. Cocaine and amphetamines.
NSAIDs are associated with a relatively modest increase in mean blood pressure of 5 mm Hg, but the increase can be more pronounced in patients with a history of hypertension controlled with antihypertensive medications.12
Glucocorticoid activity at the mineralocorticoid receptor causes sodium and fluid retention, leading to hypertension. However, synthetic glucocorticoids have less mineralocorticoid activity than cortisol, so hypertension is more prevalent in patients with endogenous Cushingâs syndrome. When the use of corticosteroid medication is unavoidable, hypertension is often responsive to dietary restrictions (eg, salt, fluid), and diuretic agents.12 Hypertension is more common in women taking oral contraceptives by a factor of 2 to 3. Risk factors include a history of gestational hypertension, family history of hypertension, cigarette use, black race, obesity, and diabetes mellitus. If the medication cannot be stopped, hypertension is generally responsive to low-dose diuretic agents.12 Common examples of sympathomimetic agents include methylphenidate, ephedrine, pseudoephedrine, oxymetazoline, and phenylpropanolamine. Some of these agents can be found in over-the-counter nasal sprays, oral decongestants, and appetite suppressants. When these agents cannot be stopped, antihypertensive therapy may be necessary. β-Blockers should be avoided because unopposed Îą-adrenergic vasoconstriction can lead to hypertensive crisis.12 For most patients, moderate alcohol intake (1 drink per day for women and 1-2 drinks per day for men) has limited effect on blood pressure. Chronic consumption of alcohol that exceeds this threshold is associated with hypertension, the severity of which follows a dose-dependent relationship. In some cases, complete abstinence from alcohol is required to control blood pressure. It is important to note that hypertension frequently develops in the setting of alcohol withdrawal, usually 2 to 3 days after the last drink.5,12-14 Tachycardia and hypertension are common manifestations of cocaine intoxication. Cocaine blocks norepinephrine reuptake in the synaptic cleft, resulting in its accumulation and subsequent activation of the sympathetic nervous system. β-Blockers should be avoided in patients with acute cocaine toxicity because unopposed Îą-adrenergic vasoconstriction can lead to hypertensive crisis and exacerbate myocardial ischemia. Nitroglycerin, calcium channel blockers, and benzodiazepines are safe and effective alternative agents.12 Acute amphetamine intoxication has a presentation similar to that of cocaine toxicity. Tachycardia and hypertension are common findings. Cocaine and amphetamines can affect the pulmonary vasculature, leading to the development of chronic pulmonary arterial hypertension.12,15
786
Other Causes of Secondary Hypertension
What are the other causes of secondary hypertension?
Hematuria and hypertension. A large neck circumference can be a clue to the presence of this condition. Discordance between blood pressure recordings in the office and those taken by the patient at home. Only women are at risk. This group of disorders is often associated with labile blood pressures, including supine hypertension and orthostatic hypotension.
What types of renal parenchymal disease are associated with hypertension?
How common is hypertension in patients with obstructive sleep apnea?
What diagnostic tool can be useful in assessing for white-coat hypertension? What is the first-line agent for pregnancy-induced hypertension? What neurologic disorders are associated with secondary hypertension?
Renal parenchymal disease related to glomerulonephritis. Obstructive sleep apnea (OSA). White-coat hypertension.
Pregnancy-associated hypertension (including gestational hypertension and preeclampsia). Neurologic disorders.
Renal parenchymal diseases associated with hypertension include chronic kidney disease (hypertension is present in the majority of these patients), acute glomerulonephritis, and scleroderma renal crisis.16 Hypertension is present in most patients with OSA. Among those without hypertension, there is a 3-fold increase in the risk of developing it. Continuous positive airway pressure (CPAP) is the most effective therapy for OSA and has been shown to improve associated hypertension.5 24-hour ambulatory blood pressure monitoring can be helpful in determining whether hypertension is situational or not. Methyldopa is first-line for pregnancy-associated hypertension. Teratogenicity must be considered before starting any medication in pregnant women.5 Neurologic disorders associated with hypertension include Increased intracranial pressure (Cushingâs response), quadriplegia, dysautonomia, and GuillainâBarrĂŠ syndrome.5
787 Case Summary A 24-year-old man with no known medical conditions presents for
evaluation of hypertension, and is found to have isolated hypertension peripheral pulses, the presence of an extra heart sound, and an of the upper extremities, bounding carotid pulses, discordant abnormal chest radiograph.
What is the most likely cause of hypertension in this patient? Coarctation of the aorta.
788
Bonus Questions
What is coarctation of the aorta? Coarctation of the aorta is a narrowing of the aorta, usually congenital in origin. Coarctation is commonly classified based on the location of the y narrowing relative to structures of the aortic arch such as the ductus arteriosus (preductal, juxtaductal, or postductal) or the left subclavian arter (proximal or distal). 17 What is the significance of the The extra sound in this case is most likely an S4 gallop associated with left ventricular hypertrophy, a common sequela of long-standing coarctation extra heart sound in this case? of the aorta. The S4 is a low-frequency late diastolic sound that is best appreciated over the apex of the heart with the bell of the stethoscope (see Figure 4-4).18 Why is coarctation of the aorta The pattern of asymmetry of the peripheral pulses in coarctationfof the aorta is dependent on the locationtof narrowing. Distal to the narrowed Where is the general location of The coarctation in this case must be distal to all of the great vessels of the aortic arch, as there are strong pulses and hypertension in the upper peripheral pulses? associated with asymmetric portion of the aorta, blood flow is compromised, whereas blood low proximal to this narrowed area is in ensified. the coarctation in this case? extremities. If the coarctation were proximal to the left subclavian artery, then blood flow to the left upper extremity would be compromised. What is the abnormal finding on The chest radiograph in this case shows ânotchingâ of the ribs (see Figure 39-1, arrows). This finding represents erosions of the bone caused by
the chest radiograph in this case? Increased pressure and blood flow through dilated intercostal arteries that have developed to supply collateral flow to the postcoarctation segment of the aorta. Rib notching is typically bilateral and affects the inferior border of the posterior third to ninth ribs. 19 What is the most frequent Patients with coarctation of the aorta commonly have coexistent bicuspid aortic valve (up to 85% of cases).20 associated with coarctation of the congenital cardiac condition aorta? What is the natural histo Coarctation of the aorta is associated with a wide range of outcomes, depending on its severity. Without correction, the mean life expectancy in coarctation of the aorta?ry of patients with coarctation of the aorta is 35 years (90% of patients die before the age of 50 years). It is associated with coronary artery disease, stroke, aortic dissection, and congestive heart failure. 20 What is the medical treatment for Medical management of coarctation of the aorta focuses on treatment for hypertension. First-line agents include β-blockers, angiotensin-converting coarctation of the aorta? enzyme inhibitors, and angiotensin receptor blockers. 20
789 Key Points
Hypertension is a key risk factor for stroke, myocardial infarction, heart failure, and kidney failure. Secondary hypertension is the abnormal elevation in systolic
arterial BP resulting from an identifiable, and often correctable, Secondary hypertension affects 5% to 10% of the general underlying cause. hypertensive population. Clinical features suggestive of secondary hypertension include the following: young age at onset, few risk factors for essential hypertension, resistant hypertension, episodes of severe
hypertension, abrupt increase in BP in a previously stable.patient, following categories: vascular, endocrinologic, toxic, and other. g labile hypertension, and evidence of target organ damage The causes of secondary hypertension can be separated into the History and physical examination are instrumental in identifyin the cause of secondary hypertension.
790
References 1. Sukor N. Secondary hypertension: a condition not to be missed. Postgrad Med J. 2011;87(1032):706-713.
- Rimoldi SF, Scherrer U, Messerli;FH. Secondary arterial hypertension: when, who, and how to screen? Eur Heart J. 2014 35(19):1245-1254.
- Berne RML, Levy MN. Physiology. 4th ed. St. Louis, Missouri: Mosby, Inc.; 1998.
-
Jennette JC, Falk RJ. The pathology of vasculitis involving the kidney. Am J Kidney Dis. 1994;24(1):130-141.
-
Chiong JR, Aronow WS, Khan IA, et al. Secondary hypertension: current diagnosis and treatment. Int J Cardiol. 2008;124(1):6-21.
-
Prisant LM, Gujral JS, Mulloy AL. Hyperthyroidism: a secondary cause of isolated systolic hypertension. J Clin Hypertens (Greenwich). 2006;8(8):596-599. Eiam-Ong S, Eiam-Ong S, Punsin P, Sitprija V, Chaiyabutr N. Acute hypercalcemia-
-
induced hypertension: the roles of calcium channel and alpha-1 adrenergic receptor. J Med Assoc Thai. 2004;87(4):410-418.
-
Magiakou MA, Smyrnaki P, Chrousos GP. Hypertension in Cushingâs syndrome. Best Pract Res Clin Endocrinol Metab. 2006;20(3):467-482.
-
Bondanelli M, Ambrosio MR, degli Uberti EC. Pathogenesis and prevalence of hypertension in acromegaly. Pituitary. 2001;4(4):239-249.
-
Colao A, Ferone D, Marzullo P, Lombardi G. Systemic complications of acromegaly: epidemiology, pathogenesis, and management. Endocr Rev. 2004;25(1):102-152. . Zuber SM, Kantorovich V, Pacak K. Hypertension in pheochromocytoma:
11characteristics and treatment. Endocrinol Metab Clin North Am. 2011;40(2):295-311, vii.
- Gyamlani G, Geraci SA.iSecondary hypertension due to drugs and toxins. South Med J. 2007;100(7):692-699; qu z 700, 8.
-
Husain K, Ansari RA, Ferder L. Alcohol-induced hypertension: mechanism and prevention. World J Cardiol. 2014;6(5):245-252.
-
Kaplan NM. Alcohol and hypertension. Lancet. 1995;345(8965):1588-1589. 15pulmonary arterial hypertension: a recent outbreak. Eur Respir Rev. 2013;22(129):244- . Montani D, Seferian A, Savale L, Simonneau G, Humbert M. Drug-induced 250.
-
Whaley-Connell AT, Sowers JR, Stevens LA, et al. CKD in the United States: Kidney Early Evaluation Program (KEEP) and National Health and Nutrition Examination
Survey (NHANES) 1999-2004. Am J Kidney Dis. 2008;51(4 suppl 2):S13-S20. 17. Nance JW, Ringel RE, Fishman EK. Coarctation of the aorta in adolescents and adults: a review of clinical features and CT imaging. J Cardiovasc Comput Tomogr.
2016;10(1):1-12. 18. Tavel ME. Clinical Phonocardiography and External Pulse Recording. 2nd ed. Chicago, Illinois: Year Book Medical Publishers, Inc.; 1967.
-
Gooding CA, Glickman MG, Suydam MJ. Fate of rib notching after correction of aortic coarctation. Am J Roentgenol Radium Ther Nucl Med. 1969;106(1):21-23.
-
Jurcut R, Daraban AM, Lorber A, et al. Coarctation of the aorta in;adults: what is the best treatment? Case report and literature review. J Med Life. 2011 4(2):189-195.
791
792